Osmosis Qs - emergency med

Pataasin ang iyong marka sa homework at exams ngayon gamit ang Quizwiz!

A 14-year-old boy comes to the emergency department because he was caught in a house fire 30 minutes ago. He is currently spontaneously breathing at a rate of 35/min and his oxygen saturations are 88% on room air. Examination shows he has some superficial and partial thickness burns on all four extremities. He also has deep burns on his forehead and occipital region. His estimated total burn area is about 75%. His nares appear inflamed, with singed hair. Which of the following is most likely the most important step in the management of this patient? A. CT head B. Cricothyrotomy C. Endotracheal intubation D. IV fluid E. Packed erythrocyte transfusion

Major takeaway Burns can occur alongside smoke inhalation injuries, and can also cause secondary airway closure due to swelling. In patients with severe burns, endotracheal intubation may therefore, be necessary. Main explanation Given his arrival at the emergency department following a house fire, and his inflamed nares and singed nasal hair, this patient is most likely a victim of smoke inhalation. He is therefore at high risk of respiratory failure from inhalation injury and should be sedated by rapid sequence and intubated. This patient has several warning signs of respiratory distress due to inhalation injury, including hypoxia, tachypnea, and singed nasal hairs. The amount of body surface area burns and the deep burns on the head are also concerning. A large number of burn patients develop airway obstruction secondary to swelling, and there is no known way to clinically predict which patients will need intubation. Hence, early intervention is necessary during inhalation injuries because there is a high risk of respiratory distress or failure. Not B. Due to risks and complications compared to endotracheal intubation, a cricothyrotomy or other surgical airway is not ideal unless a difficult airway is observed or encountered. Not D. Administering fluids is important in the management of burn patients, but it is not the primary issue in this patient at this point. Fluid administration and fluid overload also may increase the risk of respiratory failure. Not E or C. No beneficial to burn patient. CT head not indicated

A 4-year-old girl with a history of type I diabetes is brought to the emergency room in a confused and increasingly lethargic state. Her mother reports that she has been urinating more often and drinking a lot more lately. A spot urinalysis shows +4 ketones, and a finger-stick glucose test measures 300mg/dL. Which of the following exam findings would you also expect to see? A. Tachypnea / deep breathing pattern B. Dark rash in body folds C. Central obesity D. Hypothermia E. Bradycardia

Major takeaway Diabetic ketoacidosis signs and symptoms include Kussmaul breathing, tachypnea, diffuse pain, fruity odor on the breath, hyperglycemia, metabolic acidosis, and ketonuria. Main explanation This patient is experiencing diabetic ketoacidosis (DKA), an emergent complication of diabetes, most often Type I. DKA is a severe metabolic acidosis that results from insufficient insulin. Because cells cannot uptake glucose for metabolism, they begin metabolizing free fatty acids, resulting in the production of ketones. The metabolic acidosis of DKA has an elevated anion gap because of the presence of these ketones. Kussmaul breathing is one sign of DKA exhibited by tachypnea, with a deep breathing pattern. Management of DKA involves fluid repletion and administration of insulin with close monitoring of serum electrolytes. As insulin is replaced, hypokalemia can occur, as insulin also shunts potassium ions intracellularly. Potassium should be repleted as needed. Not B. Acanthosis nigricans is a brown-black velvety rash that can be seen in insulin-resistant patients, like type II diabetics. It is most commonly found in body folds. As this patient is a type I diabetic, it is highly unlikely one would see this finding Not C. Central obesity is a key finding of metabolic syndrome, a disorder of insulin resistance commonly found in type II diabetics. Patients with type I diabetes are not insulin resistant, and are more likely to be underweight. Not D. Hypothermia may be seen in the case of hypoglycemia, adrenal insufficiency, hypothyroidism, even sepsis. Diabetic ketoacidosis may be associated with fever in the case of a concomitant infection, but patients tend to remain within normal temperature limits. Not E. Patients in diabetic ketoacidosis are often dehydrated due to volume loss from excessive urination . Patients who are volume low most often present with tachycardia, dry axillae, and potentially hypotension.

A 3-day-old girl comes to the emergency department because of bilious vomiting. There was polyhydramnios detected on antenatal ultrasound. An X-ray is performed and shows a blind loop of the duodenum and gastric dilatation, which is characterized by a "double bubble" sign. Which of the following is the most likely diagnosis? A. Bochdalek hernia B. Duodenal atresia C. Gastroschisis D. Omphalocele E. Pyloric stenosis

Major takeaway Duodenal atresia is characterized by antenatal polyhydramnios and postnatal distension of the stomach with bilious vomiting. A blind loop of duodenum and gastric dilatation which presents as the "double bubble" sign can be appreciated on X-ray. Main explanation Duodenal atresia is the congenital failure of duodenum to canalize. Clinical features include antenatal polyhydramnios and postnatal distension of the stomach with bilious vomiting. A blind loop of duodenum and gastric dilatation which presents as the "double bubble" sign can be appreciated on X-ray. Treatment includes suctioning out any fluid that is trapped in the stomach, providing fluids intravenously, and surgical repair of the intestinal closure. Duodenal atresia occurs in 1 in every 5,000-10,000 live births, and 20-40% of all infants with duodenal atresia have Down syndrome. After birth, duodenal atresia may cause abdominal distension, especially in the upper abdomen. Bilious vomiting often occurs within the first couple days of life. A. Bochdalek hernia is a congenital abnormality in which an opening exists in the infant's diaphragm, allowing normally intraabdominal organs to protrude into the thoracic cavity. The lungs are often most affected, and the lung compression can be life-threatening. C. Gastroschisis is a congenital malformation of the anterior abdominal wall, which leads to exposure of the abdominal contents to the amniotic fluid. D. Omphalocele is persistent herniation of bowel into the umbilical cord due to the failure of the herniated intestines to return to the abdominal cavity during development. These contents are covered by peritoneum and amnion of the umbilical cord unlike in gastroschisis.

A 31-year-old woman comes to the clinic because of increasing abdominal discomfort for the past month. She recently returned from working on a sheep farm in Central America for the past year. Her temperature is 38.1°C (100.6°F), pulse is 62/min, respirations are 14/min, and blood pressure is 129/83 mm Hg. Physical examination shows tenderness to palpation in the right upper quadrant of the abdomen. The liver is palpable 5cm below the right costal margin. Routine laboratory studies are within normal limits. Abdominal ultrasound of the right upper quadrant (shown below) shows multiple hydatid cysts in the liver. Which of the following parasites is the most likely cause of this condition? A. Clonorchis sinensis B. Diphyllobothrium latum C. Echinococcus granulosus D. Enterobius E. Schistosoma haematobium

Major takeaway Echinococcosis is a parasitic disease caused by the *tapeworm Echinococcus granulosus*. Following the ingestion of worm eggs in *food contaminated with dog feces, individuals develop hydatid cysts, characterized by "eggshell" calcification, in the liver*. These cysts may rupture, leading to an anaphylactic reaction. Treatment includes albendazole and/or surgical removal. Main explanation Echinococcosis is a parasitic disease caused by the tapeworm Echinococcus granulosus. Following the ingestion of worm eggs in food contaminated with dog feces, individuals develop hydatid cysts, characterized by "eggshell" calcification in the liver. These cysts may rupture, leading to an anaphylactic reaction. Pharmacologic treatment includes albendazole. Surgical removal of the cysts may also be performed. The term hydatid, which is similar in etymology to the word hydrate, refers to the watery fluid inside the cysts. When the cysts are small, they are often asymptomatic, but if they grow large enough they may begin to produce pain from mass effect. Larger cysts are also at greater risk of rupture from trauma or attempted treatment, and if this occurs, the antigen-containing fluid may cause the symptoms of anaphylaxis, such as dyspnea, fever, pruritis, and edema. Also of note, sheep can be an intermediate host for this parasite, meaning that a nonproductive form of the worm can live within sheep. A. Clonorchis sinensis is a trematode that causes inflammation of the biliary tract that results in the production of pigmented gallstones and may result in cholangiocarcinoma. Transmission is from ingestion of undercooked fish B. Diphyllobothrium latum is a cestode, also called a tapeworm, that results in vitamin B12 deficiency as the worm ingests it in the intestine. This results in the development of a megaloblastic anemia. Transmission is via consumption of raw freshwater fish containing larvae. D. Enterobius is a nematode that causes anal pruritus. It's ingested via the fecal-oral route. Diagnosis is via the scotch tape test, in which tape is placed near the anus and eggs are visualized after pulling it off. E. Schistosoma haematobium is a trematode, also called a fluke, that can cause squamous cell carcinoma of the bladder and pulmonary hypertension with chronic infection. Transmission is from ingestion of undercooked fish.

A 32-year-old woman is brought to the emergency department by ambulance after having a seizure. She is 36 weeks pregnant and has no significant past medical history. Her husband says that she seized for two minutes and she reports that she has never had a seizure before. Her pregnancy has been uncomplicated thus far but she reports that she has had a headache for the last few days that has not been alleviated by acetaminophen. Physical examination shows her blood pressure is 140/100 mm Hg. Urinalysis shows 2+protein. Complete blood count, electrolytes, and liver function tests are within normal limits. Which of the following is the most likely diagnosis? A. Pre-existing hypertension B. Epilepsy C. Preeclampsia D. Eclampsia E. HELLP syndrome

Major takeaway Eclampsia and pre-eclampsia are serious complications of pregnancy. Eclampsia is new onset seizure activity in a patient with symptoms and signs of pre-eclampsia. Main explanation Eclampsia and preeclampsia are serious complications of pregnancy. Eclampsia is a complication of severe preeclampsia. Preeclampsia is characterized by new onset hypertension and proteinuria. The majority of cases occur after 28 weeks gestation in the third trimester. Patients may also report headache, blurred vision, edema, and right upper quadrant pain. HELLP syndrome (Hemolysis, Elevated Liver function tests, and Low Platelets) occurs in severe preeclampsia. If not treated preeclampsia can progress to eclampsia which is a new onset grand mal seizure or coma state in a pregnant or postpartum woman. Eclampsia is a cause of maternal death and increases the risk of fetal complications including intrauterine growth retardation, placental abruption, and premature delivery. The greatest risk to the mother is permanent CNS damage from recurrent seizures and intracranial hemorrhage.

A 20-year-old man comes to the office because of a painful forearm for the past day. He had a laceration on the same arm sutured in the emergency department 5 days ago. His temperature is 39.5°C (103°F), pulse is 110/min, respirations are 14/min, and blood pressure is 112/86 mm Hg. Examination shows his forearm has erythematous streaking and palpable crepitus. X-ray shows subcutaneous air. Which of the following is the most appropriate next step in management? A. Incision and drainage B. Direct injection of penicillin into the site of infection C. Emergent irrigation and surgical debridement of forearm D. IV epinephrine treatment E. Radiation treatment

Major takeaway Emergent irrigation and surgical debridement of forearm is the prompt treatment for this patient's condition, necrotizing fasciitis (NF). Delays in surgery are associated with a higher risk of death. Main explanation Necrotizing fasciitis (NF), commonly known as flesh-eating disease, is an infection that results in the death of the body's soft tissue. It is a severe disease of sudden onset that spreads rapidly. NF is characterized by red or purple skin in the affected area, severe pain, fever, and vomiting. The most commonly affected areas are the *limbs and perineum*. Typically, the infection enters the body through a break in the skin such as a cut or burn. Risk factors include poor immune function, obesity, alcoholism, IV drug use, and peripheral vascular disease. Between 55-80% of cases involve more than one type of bacteria. Methicillin-resistant Staphylococcus aureus is involved in up to a third of cases. The gold standard for diagnosis is surgical exploration and biopsy. *Subcutaneous air is a classic* radiographic finding associated with this diagnosis and is caused by gas-forming organisms present within the fascia. *Prevention is by good wound care and hand washing*. In this patient, NF is treated aggressively with emergent surgical irrigation and debridement. Often a combination of antibiotics are used such as penicillin G, clindamycin, and vancomycin. Delays in surgery are associated with a higher risk of death.

A 65-year-old man with a history of alcoholic cirrhosis and ascites comes to the emergency department because of seven days of worsening confusion and impaired memory. For the last two days, he has been increasingly agitated at night and spends a large portion of the daytime asleep. He denies diarrhea, melena, hematemesis, impaired balance, and double vision. He takes no medications. His temperature is 38.2°C (100.8°F), pulse is 90/min, respirations are 16/min, and blood pressure is 130/74 mm Hg. Abdominal examination shows a tender, distended abdomen with shifting dullness. The rest of the examination and a computed tomography scan of the head show no other abnormalities. Urinalysis and fecal occult blood test show no abnormalities. Serum laboratory results are shown below: Ethanol: 0.0 mg/d LLeukocytes: 12,500/mm3 Total bilirubin: 0.9 mg/dL AST: 75 U/LALT: 62 U/L Phosphatase (alkaline): 87 U/L Which of the following mechanisms is the most likely cause for this patient's suspected acute illness? A. Bacterial B. Cortical accumulation of amyloid plaques C. Hemorrhage into the subdural space D. Translocation of bacteria through intestinal wall E. Vitamin deficiency

Major takeaway Hepatic encephalopathy can be triggered by spontaneous bacterial peritonitis, which is thought to be caused by translocation of bacteria through the gut wall. Main explanation Hepatic encephalopathy is a serious complication in chronic liver disease. Once more serious causes - such as GI bleeding, intracranial hemorrhage, meningitis/encephalitis, and severe sepsis - have been excluded, a diagnosis of hepatic encephalopathy can be entertained. Clinical manifestations include acute onset of forgetfulness, confusion, altered states of arousal (e.g. lethargy, or even a flipped sleep schedule), and sometimes asterixis. Hepatic encephalopathy is suspected to occur due to increased toxin buildup in the blood caused by decreased liver function. Spontaneous bacterial peritonitis (SBP) can present with ascites and portal hypertension. Symptoms include fever, chills, nausea, and abdominal tenderness. Sometimes the only presenting symptom can be hepatic encephalopathy. A diagnostic paracentesis can rule out or confirm a diagnosis of SBP in certain patients with ascites and altered mental status. Not E. Patients with a longstanding history of alcohol consumption can become deficient in several important B vitamins. Thiamine deficiency can present as a Wernicke encephalopathy, which classically manifests as a triad of confusion, truncal ataxia, and ophthalmoplegia. It would not explain the abnormal vital signs, or his other symptoms.

The wife of a 48-year-old male patient brings him to the emergency room and says that his memory has progressively gotten worse over the last several years. She also says his personality has been changing. The physician notes abnormal writhing movements of the man's limbs and hyperreactive reflexes. MRI reveals a loss of volume in the neostriatum and cortex. In which of the following ways is this disease transmitted? A. Autosomal dominant B. Autosomal recessive C. X-linked recessive D. X-linked dominant E. Chromonosal nondssjunction

Major takeaway Huntington disease is a neurodegenerative *autosomal dominant disorder* characterized by symptoms of decreased muscular, mental, and behavioral declines. It is caused by increased *CAG repeats* gene. Main explanation This patient has Huntington disease, a neurodegenerative disorder that follows an autosomal dominant pattern of inheritance. Huntington disease is characterized by severe degeneration of the *caudate nucleus* along with degenerative changes in the *putamen and cortex*. In addition to chorea, these patients frequently suffer from athetoid (writhing) movements, progressive dementia, and behavioral disorders. The pathophysiology of Huntington disease involves an abnormally high (>40) number of CAG repeats (encodes for glutamine) in the Huntingtin gene (HTT, IT15) located at 4p16. As the altered HTT gene is passed from one generation to the next, the size of the CAG trinucleotide repeat often increases in size. A larger number of repeats is usually associated with an earlier onset of signs and symptoms. This phenomenon is called genetic anticipation. Not E. They are Downs syndrome 21 trimsomy, Turner Syndrome X monomy, Klinefelter (47, XXY)

A 36-year-old woman comes to the emergency department because of coughing, chest pain, severe shortness of breath, and fever for 9 days. She states that this is the sickest she has felt in a long time. Physical examination shows decreased lung sounds on the right side. Pneumonia is suspected, and a CT scan is obtained. A thoracentesis is also attempted, but does not produce significant drainage. Which of the following is the most appropriate next step? A. Chemical pleurodesis B. Culture of thoracentesis samples, followed by tailoring of antibiotic treatment C. placement of further chest tube

Major takeaway Pleural empyemas, which appear as dense pleural effusions on a CT scan, can develop thick rings that inhibit drainage via thoracentesis/chest tube. In these cases, surgical decortication is required. Main explanation This patient has a pleural empyema, a type of pleural effusion that is infected and has *produced pus*. These form within the pleura adjacent to the site of the infection, and have been seen in at least 40% of patients with pneumonia. While some empyemas remain liquid enough to drain from a chest tube, others form a solid rind on the external surface of the lung, compromising breathing. Treatment of pleural effusion should begin with attempted aspiration of the fluid with thoracentesis and/or chest tube drainage. If it becomes apparent that the pleural empyema is solid, surgical decortication may be required. Decortication is a surgical procedure that removes a restrictive layer of fibrous tissue overlying the lung, chest wall, and diaphragm. The aim of decortication is to remove this layer and allow the lung to reexpand. Not A. In pleurodesis, chemicals (e.g. talc) are introduced to the pleural space through a chest drain, causing irritation between the parietal and the visceral layers of the pleura. This results in adhesions and closure of the pleural space, preventing further fluid from accumulating. Pleurodesis is indicated in pneumothorax or recurrent pleural effusion, but not empyema. Not B. Cultures are usually sent in cases of pneumonia requiring hospitalization, but the causative organism of the empyema is not necessarily the same causative organism of the pneumonia. This is particularly true in adults. If antibiotics have been started, the culture will also produce few useful results. Not C. The CT scan suggests that there is a large amount of infectious material in the pleural space. If the chest tube was correctly placed, it is likely that it would have proved effective in draining some of the material. Further chest tubes will not improve drainage. Not D. While the patient has community-acquired pneumonia, the majority of the infectious material is in the pleural space. Furthermore, resolution of the infection(s) should restore lung function. Removal of the lung in this situation would not be productive in solving the infection nor beneficial to the patient.

A 40-year-old male comes to the emergency department by EMS with suspected overdose and respiratory depression. EMS found the man laying on his couch with snoring respirations, completely unresponsive. His initial vital signs are significant for a respiratory rate of 5 breaths per minute. Physical examination reveals pinpoint pupils, and track marks along both arms. Which of the following is the next best step in the management of this patient following airway stabilization? A. Substance abuse consult B. Methadone C. Naloxone D. Flumazenil

Main explanation Naloxone is an opioid receptor antagonist used for treating opioid overdose. Patients may require multiple doses of naloxone depending upon the dose and potency of opioids causing the overdose. In particular, batches of heroin laced with fentanyl can be particularly resistant to naloxone antagonism. Major Takeaway: Suspect opioid overdose in a patient with decrease respirations, pinpoint pupils, and a history or evidence of drug abuse. Treatment is with an opioid receptor antagonist such as naloxone. Flumazenil is indicated for a benzodiazepine overdose.

A 16-year-old girl is hospitalized after a dirt-bike accident. She has a spiral fracture of the distal tibia and several broken ribs. She is given an IV medication for pain control. Soon after, she experiences intense pruritus and breaks out in a rash. Which answer choice best describes the difference in urticaria precipitated by non-opiate pain medications or penicillin versus urticarial lesions triggered by opiates? A. Opiate-triggered reaction NOT involve IgE B. Opiate-triggered reaction NOT blanch with pressure C. Opiated-administered most often triggered by oral administration D. Opiate-triggered reaction does not demonstrate dermographism

Main explanation Opiates like morphine are most classically associated with urticaria, but opioids like oxycodone may trigger it as well. These reactions, considered to be "anaphalactoid," are believed to occur by direct attachment of drug particles to mast cells stimulating histamine release, *without elevations in serum IgE* because it is not "allergic reaction". As such itching caused by opioids are more akin to an adverse reaction rather than a hypersensivity reaction. *Co-administration of antihistamines can help to mitigate the effects of histamine.* Major Takeaway: Opiates and opioids frequently cause itching, which is believe to be due to a direct effect on mast cells. Anti-histamines can relieve itching symptoms. B. Lesions that blanch with pressure are characteristic of ALL urticarial rashes. C. Oral codeine derivatives may cause histamine-release with urticaria and pruritus. However, urticaria in response to opiates most commonly occurs with intravenous administration. D. Dermographism (wheals that appear with light scratching) is generally characteristic of all urticarial rashes.

A 45-year-old man is brought to the emergency department because of chest pain for the past 3 hours. The patient has been seen several times in the last year for similar symptoms, however, each time cardiac evaluation is normal. History reveals no cardiac risk factors. The patient says he barely leaves his house and had to quit his job because he fears his symptoms of chest pain will begin and he will not be able to reach a hospital. After thorough evaluation, all tests are negative. The patient becomes frustrated and storms out of the hospital demanding a second opinion. Which of the following is the most likely diagnosis? A. Adjustment disorder B. Conversion disorder C. Somatic symptom disorder D. Panic disorder E. Post-traumatic stress syndrome

Main explanation Somatic symptom disorder is a DSM-V diagnosis characterized by excessive preoccupation with minor bodily symptoms. Patients often self diagnose themselves and do not believe the physician's negative work-up. Preoccupation with or fear of their symptomatology can often inhibit a patients ability to perform important activities, such as work, school activities, or family and social responsibilities. The preoccupation must be present for at least six months. Treatment consists of regular follow-up visits and offering psychotherapy. The patient should not be told that these symptoms are imaginary. This diagnosis was previously known as hypochondria, but in the DSM-V that diagnosis is not possible. A similar DSM-V diagnosis to somatic symptom disorder is illness anxiety disorder which is characterized by intense anxiety about an undiagnosed condition.The media and the Internet often contribute to these disorders through articles, TV shows, and advertisements regarding serious illnesses. Inaccurate portrayal of risk and the identification of non-specific symptoms as signs of serious illness contribute to exacerbating the hypochondriac's fear that they actually have that illness. A. Adjustment disorder is the term used to describe a period of emotional symptoms (such as anxiety or depression) that cause impairment following an identifiable psychosocial stressor. These symptoms commonly last less than six months but may occur for an extended period of time in presence of chronic stressor. B. Conversion disorder is a diagnostic category previously used in some psychiatric classification systems. It is sometimes applied to patients who present with neurological symptoms , such as numbness, blindness, paralysis, or fits, which are not consistent with a well-established organic cause, and which cause significant distress.

A 48-year-old man comes to the emergency room because of severe, burning epigastric pain . He has had similar pain refractory to multiple trials of different proton pump inhibitors for the past 6 months. An outpatient upper endoscopy 2 months ago revealed prominent fundal gastric folds and ulcerations of the 1st and 2nd parts of the duodenum. He was consequently treated with triple therapy for presumed H. pylori infection. Despite treatment he continues to experience significant daily pain. He also notes intermittent diarrhea for the past 4 months. Of late he has noticed that his pants are fitting more loosely than normal. Physical examination is normal except for some mild tenderness in the epigastric region. Laboratory studies shows a serum gastrin of 1136 pg/mL. Which of the following is the most likely diagnosis for this patient?

Major Takeaway: Zollinger-Ellison syndrome results from the oversecretion of gastrin by a neuroendocrine tumor. It usually presents as gastrointestinal ulcerations refractory to acid-suppressing treatments (PPI) and is diagnosed by measuring serum gastrin levels. Zollinger-Ellison syndrome (ZES) results from the presence of a gastrin-secreting tumour. Excess gastrin leads to hypersecretion of gastric acid and the development of peptic ulcers. It most commonly presents with abdominal pain and diarrhea. A serum gastrin level > 1000 pg/mL coupled with a gastric pH < 2.0 is diagnostic for ZES. However, further confirmatory testing is required and an acid output test (basal & maximal) can be used for this purpose. ZES is expected in patients with, ulceration in unusual locations (e.g. 2nd part of the duodenum), peptic ulcer disease that is refractory to acid-suppressing treatment and/or treatment for H. pylori, and ulcers with hypercalcaemia. Patient often have personal or family history of multiple endocrine neoplasia (MEN) 1 syndrome [the Three P's: tumors of the Pancreas (e.g. gastrinoma), Parathyroid, and Pituitary].

A 70-year-old man is brought to the emergency department because of slurred speech, confusion, and weakness in his right side for the past hour. He has complete hemiparesis of his right upper and lower extremities. Patient's medical history is significant for hypertension, atrial fibrillation, and diabetes for the past 26 years. His temperature is 37.8°C (100°F), pulse is 100/min, respirations are 14/min, and blood pressure is 150/92 mm Hg. Oxygen saturation is 94% in room air. Physical examination shows a right-sided facial droop to the lower portion of his face, symmetric forehead wrinkles, and symmetric eyebrow elevation. Which of the following is the most likely diagnosis? A. Acute hemorrhaggic stroke B. Acute ischemic stroke C. Bell palsy D. Complex hemiplegic migraine E. Subarachnoid hemorrhage

Major takeaway Acute ischemic stroke (AIS) accounts for 80% of all cerebrovascular accidents. AIS is from embolism or local thrombosis and is characterized by an acute onset of any focal neurological findings. Brain CT scan, for evaluating hemorrhagic infarct or subarachnoid hemorrhage, is necessary. Main explanation Ischemic stroke is characterized by the sudden loss of blood circulation to an area of the brain, resulting in a corresponding loss of neurological function. Acute ischemic stroke is caused by thrombotic or embolic occlusion of a cerebral artery and is more common than hemorrhagic stroke. Acute ischemic stroke accounts for 80% of all CVAs, and is from embolism or local thrombosis. CT scan is used for evaluation of early hemorrhagic infarct or subarachnoid hemorrhage. Consider stroke in any patient presenting with acute neurological deficit or any alteration in level of consciousness. Common stroke signs and symptoms are abrupt onset of hemiparesis, monoparesis, or (rarely) quadriparesis, hemisensory deficits, monocular or binocular visual loss, visual field deficits, diplopia, dysarthria, facial droop, ataxia, vertigo, nystagmus, aphasia, and sudden decrease in level of consciousness. The goal for the emergent management of stroke is to complete the following within 60 minutes of patient arrival: assess airway, breathing, and circulation (ABCs) and stabilize the patient as necessary, complete the initial evaluation and assessment, including imaging and laboratory studies, and initiate re-perfusion therapy, if necessary. Ischemic stroke therapies include fibrinolytic therapy, antiplatelet agents, and mechanical thrombectomy.

A 20-year-old man is brought to the emergency department by ambulance because of hemoptysis and respiratory failure requiring ventilatory support. Temperature is 36.8°C (98°F), pulse is 105/min, respirations are 28/min, and blood pressure is 117/78 mm Hg. Examination shows a distressed appearing patient wearing a non-rebreather mask. Urinalysis shows microscopic hematuria, proteinuria, and red cell casts. Serologic assays for anti- glomerular basement membrane (anti-GBM) antibodies are positive. Which of the following is the most likely diagnosis? A. Churg-Strauss syndrome B. Goodpasture syndrome C. Granulomatosis with polyangiitis D. Rheumatoid arthritis E. Pulmonary embolism

Major takeaway Goodpasture syndrome is one of the causes of pulmonary renal disease (pulmonary haemorrhage with rapidly progressive glomerulonephritis). Prompt diagnosis is key and can be achieved with renal biopsy and serological testing. Main explanation The most likely diagnosis for this patient is Goodpasture syndrome . Goodpasture's syndrome classically presents with the triad of pulmonary hemorrhage, rapidly progressive glomerulonephritis and anti-glomerular basement membrane (anti-GBM) antibodies. These antibodies can either be in circulation or fixed to the kidney. In the appropriate clinical setting (ie, alveolar hemorrhage and urinary findings suggestive of an acute glomerulonephritis), the detection of circulating anti-glomerular basement membrane (anti-GBM) antibodies allows the clinician to make a firm diagnosis of anti-GBM disease. The three principles of therapy in anti-glomerular basement membrane (anti-GBM) disease are as follows: Rapidly remove circulating antibody, primarily by plasmapheresis Stop further production of antibodies using immunosuppression with medications Remove offending agents that may have initiated the antibody production Patients who develop massive hemoptysis or acute respiratory failure (such as the patient in this question) should be cared for in an intensive care unit (ICU). Transfer to a hospital where plasmapheresis and/or hemodialysis is available may be necessary. Churg-Strauss syndrome can cause rapidly progressive glomerulonephritis , but is associated with a history of asthma, atopy, or allergic symptoms. Anti-glomerular basement membrane (anti-GBM) antibodies are negative.

A 26-year-old pregnant woman comes to the emergency department because of chronic pelvic pain. This is the second time the patient has visited the emergency department within the last month. According to her electronic medical records, her previous vital signs were normal and her physical examination showed bruising around the right eye. Her temperature is 37°C (98.6°F), pulse is 85/min, respirations are 16/min, and blood pressure is 129/76 mm Hg. Physical examination shows bruises on her abdomen. Which of the following is the next appropriate next step in management? A. Asking leading questions B. Do not ask about the bruises C. Document the injuries D. Order CT abdomen and pelvis E. Verbally screen for domenstic abuse in the presence of her partner, family member, or friend

Major takeaway A substantial number of pregnant women are victims of domestic violence. Women who are battered during pregnancy are more likely to seek healthcare for injuries than women battered before pregnancy. It is essential for the physician to know how to deal with cases like these. Main explanation A substantial number of pregnant women are victims of domestic violence. Women who are battered during pregnancy are more likely to seek healthcare for injuries than women battered before pregnancy. It is essential for the emergency physician to know how to deal with cases like these. As a medical provider your role in the screening and intervention process is to: - Screen for abuse - Assess the medical context of the abuse (e.g. the severity and frequency of injuries; whether food or medication are being withheld from the patient) - Provide medical treatment for the presenting injuries - Assess the safety of the victim - Briefly counsel the victim regarding available resources and management options - Document your findings and referrals. - Take photos of the injuries - Refer the patient to relevant multidisciplinary colleagues

A 58-year-old man is brought to the emergency department because of severe lower back pain which radiates to his right flank and groin. He is alert and oriented on arrival to the hospital, but soon begins to be difficult to arouse. Physical examination shows an afebrile, cyanotic, tachycardic, and hypotensive patient. Abdominal examination shows a pulsatile mass in the right flank. Which of the following is the most likely diagnosis? A. Acute pancreatitis B. Acute renal injury C. Complicated appendicitis D. Ruptured abdominal aortic aneurysm E. Sigmoid volvulus

Major takeaway Abdominal aortic aneurysm rupture is most commonly characterized by abdominal or back pain and a pulsatile abdominal mass. It is a focal dilation of the vessel wall >50% of the normal diameter. Main explanation Abdominal aortic aneurysm rupture is a medical emergency most often characterized by abdominal, flank, or back pain, hypotension, and a pulsatile abdominal mass. An aortic aneurysm is a focal dilation of the vessel wall >50% of the normal diameter, and occurs due to degeneration or tear of vessel wall components, usually the tunica media. Aneurysms can occur at various points of the aorta and are classified by location. Risk factors for the development of an aortic aneurysm include age, hypertension, and connective tissue disorders. Aortic aneurysms are usually asymptomatic until they rupture. When they do, surgical intervention is required to repair the rupture. >60% of patients with a ruptured abdominal aortic aneurysm die of cardiovascular collapse. Therefore, it is crucial to diagnose an abdominal aortic aneurysm early to prevent complications. *Ultrasonography is the preferred method of choice for the diagnosis of an abdominal aortic aneurysm due to ease of access and bedside performance*. Other diagnostic imaging tools include radiography, computed tomography, and magnetic resonance imaging.

A 33-year-old basketball player comes to the office because of severe pain in his right ankle for 20 minutes. He says that he was running to make a basket and when he stopped and turned to shoot the ball, he felt as though someone struck the back of his ankle "with a sledgehammer." He also heard a "pop." Examination shows the patient is able to walk, although he is limping. He is able to dorsiflex and plantar flex both feet. With the patient kneeling on a chair, squeezing the right gastrocnemius muscle belly does not result in plantar flexion. Which of the following is the most likely diagnosis? A. Achilles tendon rupture B. Ankle fracture C. Ankle sprain D. Enthesopathy E. Retrocalcaneal bursitis

Major takeaway Achilles tendon rupture usually occurs while performing tasks that *require fast acceleration* such as jumping, running, or pivoting. Patients describe a sensation of being "kicked behind the ankle" associated with hearing a loud "pop". Following the injury, pain and swelling are common. *The Thompson ("calf-squeeze") test will be positive* Main explanation Pivoting and jumping are both associated with rupture of the achilles tendon. Pain in achilles tendon injury tends to occur 2 to 6cm proximal to the tendon's insertion (red rectangle on image). Keep in mind that, even with an achilles tendon rupture, patients may be able to walk and dorsiflex/plantar flex the ankle. The test performed in the question is called the Thompson Test. *Squeezing the gastrocnemius muscle should lead to plantar flexion*. When it does not, this is a positive Thompson test. In one study of 174 patients, the test was found to have over 90% sensitivity and specificity for tendon rupture. The site of pain in the posterior ankle may help you to narrow your differential diagnosis. With heel pain, you may want to consider bursitis, enthesopathy, and plantar fasciitis. The pain associated with achilles tendonitis or achilles tendon rupture is higher up. Achilles tendon rupture may be obvious on clinical exam, but ultrasound can also be useful to confirm the diagnosis.

A 55-year-old man comes to the emergency department because of blunt trauma to his left forearm from heavy equipment. He developed mild pain and swelling but did not seek any medical attention. Today, the patient developed severe pain, swelling, and numbness in his left forearm. His temperature is 37.1°C (98.8°F), pulse is 80/min, respirations are 16/min, and blood pressure is 130/76 mm Hg. Physical examination of the left upper limb shows tense swelling in the left forearm and tenderness that was exaggerated by extension of the fingers. His radial pulse is intact. Which of the following is the most likely diagnosis? A. Acute compartment syndrome B. Brachial arterial injury C. Fracture of radial bone D. Volkmann contracture

Major takeaway Acute compartment syndrome occurs when the tissue pressure within a closed muscle compartment exceeds the perfusion pressure and results in muscle and nerve ischemia. Patients with compartment syndrome are characterized by pain out of proportion to the injury. Main explanation Acute compartment syndrome occurs when the tissue pressure within a closed muscle compartment exceeds the perfusion pressure and results in muscle and nerve ischemia. It typically occurs subsequent to a traumatic event, most commonly a fracture. The patient usually complains of pain out of proportion to the injury. The pain usually worsens with passive stretching of the muscles affected. The traditional 6 P's of acute ischemia in a limb (i.e, pain, paresthesia, paralysis, pallor, pulselessness, poikilothermia) are not clinically reliable, but can be clues to diagnosis. These signs may manifest only in the late stages of compartment syndrome, by which time extensive and irreversible soft tissue damage may have taken place. Peripheral pulses and capillary refill can be normal in cases of upper extremity acute compartment syndrome. The award-winning science journalist and former CNN correspondent and anchor, Miles O'Brien had this condition. In February 2014, O'Brien was injured when a case fell on his left forearm, causing acute compartment syndrome and resulting in the amputation of his left arm above the elbow. Not C. In radial bone fractures, the patient is characterized by the development of severe pain since the time of trauma with deformity and restriction of movement. Not D. Volkmann contracture is a late complication of compartment syndrome. The prolonged ischemia in the compartment results in permanent deformity and contracture of the hand. The radial pulse is usually absent by the time this deformity occurs.

A 55-year-old woman comes to the emergency department because of increased dyspnea and sputum production for the past day. She reports that her cough and dyspnea is worse than what it is on a regular daily basis. Her daughter recently had an upper respiratory infection and she believes she is coming down with the "common cold." She has a history of emphysema and a 30 pack year history of smoking. Pulmonary auscultation shows bibasilar wheezing. Pulse oximetry shows an oxygen saturation level of 90%. The woman suddenly coughs and produces purulent sputum. Which of the following is the next best step in management? A. Albuterol, corticosteroids, and antibiotics B. IV antbiotics C. Mechanical ventilation D. Nitrates and aspirin E. Oxygen therapy via re-breather mask

Major takeaway Acute exacerbation of chronic obstructive pulmonary disease is characterized by increased dyspnea, cough, and sputum production that is worse than normal. Treatment includes a short-acting beta-2 agonist and systemic corticosteroids. In certain situations, antibiotics, oxygen therapy, and mechanical ventilation should be considered. Main explanation Chronic obstructive pulmonary disease (COPD) is an obstructive process leading to air trapping within the lung cavity. Pulmonary function testing will show elevated residual volume and a decreased forced expiratory volume in 1 second and functional vital capacity ratio. Examples of COPD include chronic bronchitis, bronchiectasis, and emphysema. The major risk factor for COPD is smoking. However, COPD can be secondary to genetic conditions (such as alpha-1 antitrypsin deficiency) as well.Managing an acute exacerbation of COPD is important in order to prevent morbidity and mortality in these patients. These exacerbations are characterized by increased dyspnea, cough, and sputum production that is worse than normal. While many times idiopathic, an exacerbation can be caused by an upper respiratory infection. Pulse oximetry and arterial blood gas should be done to assess severity. Treatment of an acute exacerbation includes a short-acting beta-2 agonist such as albuterol and systemic corticosteroids. Antibiotics should be given to patients in certain cases of worsening dyspnea and sputum production. Oxygen therapy should be initiated in cases in which the patient PaO2 is <55 mmHg or with a pulse oximetry measurement of M88%. In severe cases, mechanical ventilation should be initiated.

A 60-year-old man comes to the office due to a diffuse, pruritic rash and generalized lymphadenopathy. He states that the rash started across the lower half of his body four months ago and has since spread across most of his body. He began noticing the lymphadenopathy about one week ago. Physical examination shows generalized erythroderma sparing only the left foot and the right half of the face. The liver and spleen are enlarged on palpation. A complete blood count with differential is collected and shows the following values:Erythrocytes: 3.9 million/mm3Platelets: 180,000/mm3Leukocytes: 12,000/mm3Segmented neutrophils: 42%Eosinophils: 0%Basophils: 0%Monocytes: 1%Lymphocytes: 57%A peripheral blood smear is obtained and shown below. Which of the following is the most likely diagnosis? A. Adult T cell leukemia-lymphoma B. Hairy cell leukemia C. Langerhans cell histiocytosis D. Mycosis fungoides E. Sezary syndrome

Major takeaway Cutaneous T cell lymphomas include mycosis fungoides and Sezary syndrome. Of the two, Sezary syndrome is more likely to manifest with widespread erythroderma and the presence of atypical T cells with cerebriform nuclei on peripheral blood smear. Main explanation Mycosis fungoides (MF) and Sezary syndrome (SS) are cutaneous T cell lymphomas, both involving mature CD4+ T lymphocytes that localize to the skin. The main difference between the two non-Hodgkin lymphomas is the degree of involvement. *If only the skin is involved, a diagnosis of MF is more likely. SS generally refers to lesions that have spread beyond the skin to the peripheral blood.* Patients with SS are thus more likely to experience lymphadenopathy, hepatosplenomegaly, and bone marrow involvement.While the lesions in MF commonly start out as patches or plaques and tend to slowly progress, patients with SS develop diffuse lesions early on. The most common skin lesion is erythroderma, diffuse reddening of the skin which may be covered by scale. A body surface area of >80% must be involved for a diagnosis of SS to be made. Pautrier microabscesses, collections of neoplastic cells in the epidermis, can be seen in both disorders. The characteristic cell SS is a Sezary cell, atypical T cells with a characteristic grooved, "cerebriform" nucleus, seen in a peripheral blood smear. A. Adult T cell leukemia-lymphoma (ATL) is a hematologic neoplasm of CD4+ T cells caused by the human T-lymphotropic virus, type 1 (HTLV-1). Features include generalized lymphadenopathy, hepatosplenomegaly, hypercalcemia, and malignant cells in peripheral blood. Skin lesions can be similar to those in MF/SS, with 4% of patiens having erythroderma. B. Hairy cell leukemia (HCL) is a neoplasm of mature B cells that stain positive for tartrate-resistance acid phosphatase (TRAP) and are visualized with filamentous, "hairy" projections. HCL is characterized by splenomegaly and myelofibrosis, yielding a "dry tap" on bone marrow aspiration. Lymphadenopathy is uncommon. C. Langerhans cells are specialized dendritic cells localized to the skin. Langerhans cell histiocytosis (LSH) refers to a neoplastic proliferation of Langerhans cells, which can be identified with staining for CD1a and S100. They also have characteristic intracellular Birbeck granules, shaped like tennis racquets. D. Mycosis fungoides is a neoplasm of mature CD4+ T cells localized to the skin. Skin lesions include patches, plaques, tumors, and erythroderma. Pautrier microabscesses, aggregates of neoplastic cells, are pathognomonic. Malignant cells with cerebriform nuclei can be seen on skin biopsy.

A 70-year-old man is brought to the emergency department because of left-sided weakness for one hour. The patient reports that he suddenly could not move his left leg at home. The patient denies any history of a bleeding disorder, active bleeding, or recent surgery. His past medical history is significant for hypercholesterolemia, hypertension, atrial fibrillation, and tobacco abuse. Physical examination shows 5/5 strength in the left upper extremity and 1/5 strength in the left lower extremity. There is decreased sensation in the left leg and foot. A computed tomography scan of the brain shows no hemorrhages. Which of the following is the next best step in management? A. Alteplase B. Aspirin C. Clopidogrel D. Unfractionated heparin E. Warfarin

Major takeaway Altepase is a tissue plasminogen activator (tPA). These medications should be given to patients with symptoms of stroke within 3-4.5 hours of initial onset. Main explanation Stroke and other cerebrovascular incidents are the third leading cause of death. Causes of stroke include atherosclerosis, hemorrhage, and thrombotic emboli. Hence, for this reason, patients who have a history of atrial fibrillation require continuous anticoagulation therapy. Symptoms of stroke include facial droop, slurred speech, paralysis, muscle weakness, sensation loss, and confusion. First step in management is to perform a computed tomography (CT) scan of the head. If no bleeding is present, then the patient may be a candidate for tissue plasminogen activator (tPA). Contraindications include recent stroke, recent intracranial or spinal injury, history of intracranial hemorrhage, active bleeding, active intracranial hemorrhage, blood pressure greater than 185/110 mmHg, arteriovenous malformation, and intracranial neoplasm. tPA needs to be given within 3-4.5 hours of symptom onset. Altepase is a tPA which activates plasminogen in order to break up clots. It is effective in one-third of cases, has no effect in one-third, and in one-third of cases can lead patients to get worse. If effective, it can provide complete resolution of symptoms rapidly.

A 45-year-old man is wheeled into the operating room for a surgical evacuation of an epidural hematoma. Which of the following hemodynamic changes would be indicators concerning for an increase in intracranial pressure (ICP)? A. Hypotension and tachycardia B. Hypertension and bradycardia C. Hypertension and tacyhcardia D. Hypotension and bradycardia

Major takeaway An increase in intracranial pressure leads to Cushing's reflex, the result of which is often described as *Cushing's triad which comprised of: hypertension, bradycardia, and irregular respiration.* Main explanation The correct answer to this question is hypertension and bradycardia. Cushing's triad describes the symptoms observed due to Cushing's reflex, the sympathetic/parasympathetic response to increased intracranial pressure.

A 35-year-old woman comes to the emergency department because of progressively worsening right upper abdominal pain for two days. The patients states that for the past 3 months she's had a similar pain 1-2 hours after meals, but now the pain is more severe and constant. She admits to fever, dark-colored urine, and clay-colored stools. She denies pain radiation, nausea, or vomiting. Physical examination shows a middle aged female in moderate distress with mild jaundice. Abdominal examination shows tenderness to palpation in the upper right quadrant. A pregnancy test is negative. A complete blood count shows leukocytosis and elevated alkaline phosphatase. Blood culture is positive for Escherichia coli. Based on the patient's presentation, which of the following is the most likely diagnosis? A. Cholecystitis B. Ascending cholangitis C. Biliary colic D. Choledocholithiasis E. Chronic pancreatitis

Major takeaway Ascending cholangitis (also called acute cholangitis) is an ascending bacterial infection of the biliary tract thought to be related to choledocholithiasis and bile stasis. It is characterized by Charcot's triad of upper right quadrant pain, fever, and jaundice. Main explanation The patient in this question most likely has ascending cholangitis as evidenced by Charcot's triad of right upper quadrant pain, fever, and jaundice. Ascending cholangitis (also called acute cholangitis) is an ascending bacterial infection of the biliary tract thought to be related to choledocholithiasis and bile stasis. If left untreated, this condition has high potential for morbidity and mortality. Typically, patients may have a *history of choledocholithiasis or recent biliary tract manipulation*. In a more severe form of the disease, septic shock (hypotn or tachycardiac) and mental confusion can also be present and these symptoms together with Charcot's triad are called Reynolds pentad. Physical examination findings include jaundice, icterus, fever, and abdominal pain. Laboratory tests typically show leukocytosis, hyperbilirubinemia, elevated alkaline phosphatase, elevated transaminase and serum amylase, and blood cultures are positive in almost 50% of patients. The top isolated microorganisms include E. coli, Klebsiella, Enterococcus spp, and Bacteroides fragilis. Once diagnosed, administration of broad-spectrum intravenous antibiotics and correction of fluid and electrolyte imbalances are the recommended treatments. Not C. Biliary colic describes pain that occurs due to a gallstone temporarily blocking the bile duct. The pain can last for a few hours but typically subsides. Not D. Choledocholithiasis refers to a stone in the common bile duct. It can cause jaundice and liver damage, but does not typically present with fever. Not E. Chronic pancreatitis often presents with abdominal pain that can radiate to the back and can occur after or independently of meals.

A 40-year-old man comes to the emergency department because of bilateral facial weakness during the past 3 days. He also reports a change in taste sensation. One month ago, he had flu-like symptoms after returning from a climbing trip in New York. Physical examination shows flattening of the skin of his forehead. Motor examination shows difficulty with facial movements bilaterally, including smiling and raising his eyebrows. The remainder of the examination shows no abnormalities. Which of the following is the most likely diagnosis? A. Amyotrophic lateral sclerosis B. Diabetes mellitus C. Lyme disease D. Systemic Lupus erythematous E. Multiple sclerosis

Major takeaway Bilateral facial nerve palsy is often associated with untreated Lyme disease. Unilateral palsy is often idiopathic and may be associated with herpes zoster infection. Main explanation This patient's symptoms of facial weakness, loss of wrinkling of the forehead, and change in taste sensation are consistent with a *bilateral facial nerve (cranial nerve VII) palsy*. Bilateral facial nerve palsy is rare compared to unilateral involvement and is most *commonly associated with Lyme disease and Guillain-Barré syndrome*. Patients typically have a sudden onset of symptoms, which *can be concerning for a stroke*; however, most patients will make a full recovery, with signs of recovery seen within 3 weeks. Lyme disease is a tick-borne illness caused by the bacteria Borrelia burgdorferi. The disease is endemic to the northeastern United States and is often seen in hikers or others who spend time outdoors. Patients may report an initial flu-like syndrome that often goes ignored. The classic sign is *erythema migrans, a "bulls-eye" rash with central clearing*. If untreated, cardiac and neurologic manifestations, including bilateral facial nerve palsy, may occur. B. Amyotrophic lateral sclerosis is a neurological disorder that causes progressive painless muscle weakness, typically in the extremities. Exclusive involvement of facial muscles is not typical. D. While clinical signs could varies, SLE should not cause bilateral facial palsy

A 50-year-old woman comes to the emergency department with severe colicky abdominal pain that has worsened over the past 48 hours. She has a past history of endometriosis and a surgical history of appendectomy and a cesarean section. Physical examinations shows a distended abdomen that is tympanitic to percussion. An abdominal radiograph is obtained and is shown below. Which of the following is the most likely diagnosis? A.Constipation B. Ovarian torsion C. Pelvic inflammatory disease D. Small bowel obstruction E. Volvulus

Major takeaway Bowel obstruction occurs most commonly in the small bowel, usually due to post-surgical adhesions. Presentation includes colicky abdominal pain, distended abdomen, a history of abdominal surgery, an abdomen that is tympanitic to percussion and has high-pitched bowel sounds. Main explanation Bowel obstruction most commonly occurs in the small bowel. Typical symptoms include severe colicky abdominal pain, abdominal distention, nausea and vomiting, fever and tachycardia. Most patients will have a history of prior abdominal surgery. Physical examination will show a lack rebound tenderness, be tympanitic to percussion, and have high-pitched bowel sounds. Radiologic studies generally show bowel distention, air-fluid levels, and an absence of air distal to the obstruction. There are many causes of bowel obstruction including adhesions, hernia, Crohn disease, malignancy, atresia, intussusceptions, and volvulus. Adhesions are the most common cause of bowel obstruction and can form post-operatively from inflammation of the bowel wall. Initial treatment includes fluid resuscitation, bowel decompression with a nasogastric tube, analgesics, and surgical consultation. Small bowel obstruction with signs of ischemia, necrosis, or perforation is a surgical emergency. Obstruction that does not resolve with initial treatments also requires surgery.

A 32-year-old pregnant woman, gravida 1, para 0, in her first trimester comes to the clinic because of increasing abdominal girth over the past two weeks. She has associated right upper quadrant abdominal pain without radiation. She has not had any pelvic pain or discharge. Her past medical history shows factor V Leiden mutation. She has no family history of cardiac or liver disease. Recent first trimester laboratory tests for gonorrhea, chlamydia, syphilis, and HIV were all negative. Hepatitis B titers were negative for hepatitis B surface antigen, hepatitis B core antigen, and hepatitis B core antibody, but positive for hepatitis B surface antibody. She denies any history of alcohol or intravenous drug use. Her vital signs show no abnormalities. Physical examination shows jaundice, non-tender hepatomegaly with abdominal distension, and ascites. Her neck veins appear flat. Which of the following is the most likely diagnosis? A. Budd-Chiari syndrome B. Cholangitis C. Fitz-Hugh-Curtis syndrome D. Spontaneous perforation of bile duct E. Toxoplasmosis

Major takeaway Budd-Chiari syndrome is an uncommon condition caused by hepatic venous outflow tract obstruction. Characteristic symptoms include hepatomegaly, ascites, and right upper quadrant abdominal pain, with or without jaundice. It is associated with polycythemia vera, thrombophilic conditions (such as factor V Leiden), pregnancy, oral contraceptive use, and vena cava abnormalities. Main explanation Budd-Chiari syndrome is caused by hepatic venous outflow tract obstruction, often thrombotic. Characteristic symptoms include hepatomegaly, ascites, and right upper quadrant abdominal pain, with or without jaundice. It is most common in patients with underlying hypercoagulability, including conditions such as pregnancy, cancer, chronic inflammatory disease, clotting disorder (such as factor V Leiden mutation), infection, or myeloproliferative disorder (such as polycythemia vera). The chronic form is most common, characterized by progressive ascites and possible renal impairment. The acute form often results in hepatic and renal failure. Treatment is aimed at alleviating the obstruction, such as via anticoagulant or thrombolytic therapy. In severe cases, Budd-Chiari is an indication for liver transplantation. Not C. This is a rare complication of pelvic inflammatory disease, usually secondary to infection with gonorrhea and/or chlamydia. The inflammation from these infections creates scar tissue on Glisson's capsule, the connective tissue surrounding the liver, often causing acute onset right upper quadrant pain or pleuritic chest pain. Not D. Seen in kids Not E. While toxoplasmosis is more common in immunosuppressed individuals, the immunosuppression in pregnancy is not notable until third trimester. Acute infection is typically asymptomatic; however, non-specific flu-like symptoms such as fever, fatigue, headache, and myalgias may be present. Fetal sequelae include damage to the eye (chorioretinitis) or brain (hydrocephalus, intracranial calcifications).

A 19-year-old man comes to the emergency department after being rescued from a burning building 20 minutes ago. His right lower leg was trapped under a burning beam, resulting in a severe full-thickness and circumferential burn. He sustained no other injuries and he is currently stable on room air. In the ED, he begins feeling severe pain in his right foot and toes. Examination shows there is increased edema of the lower leg, pain with passive movement of the toes, and subjective "tingling" in the right foot. Dorsalis pedis and posterior tibial pulses are palpable bilaterally. There is no cyanosis. Which of the following is the most appropriate next step in management? A. Amputation B. Emergency escarotomy C. Obtain an X-ray D. Prescribe analgesic E. Reassure patient

Major takeaway Burns of a circumferential nature are at risk for eschar development and therefore limb compartment syndrome. Treatment is with escharotomy, to relieve the intercompartmental pressure and ensure adequate limb perfusion. Main explanation This patient has compartment syndrome, which occurs as a result of compression of the neurovascular structures due to increased pressures from soft tissue swelling and the unyielding fascial sheath. He needs an emergency escharotomy to release the pressure. Full-thickness or near full-thickness skin burns lead to development of a tough, inelastic mass of burnt tissue known as an eschar. Circumferential eschars can lead to burn-induced compartment syndrome where soft tissue swelling from the accumulation of fluid within confined fascial compartment of the extremities increases the pressures surrounding neurovascular structures. This compression of vascular and lymphatic structures lead to decreased perfusion, accumulation of metabolic waste, and can cause tissue death. Remember the 6 P's! Pain - especially pain on passive movement (out of proportion to injury)Paresthesias (hyperesthesia, numbness - decreased sensation and 2 point discrimination)Pallor - or cyanosisPoikilothermia - cold feeling extremityParalysis - late, ominous signPulselessness - lack of palpable pulses is an ominous, late sign

A 40-year-old man comes to the emergency department because of a 3-week history of episodic cutaneous flushing, diarrhea, and wheezing. He has a past medical history of hypertension and type 2 diabetes mellitus. His temperature is 36.6°C (97.9°F), pulse is 125/min, respirations are 30/min, and blood pressure is 90/60 mm Hg. Pulmonary examination shows diffuse wheezes in both lung fields. Cardiac examination shows a prominent "v" wave of the jugular vein and a 1/6 holosystolic murmur best heard on the left lower sternal border. Abdominal examination shows hyperactive bowel sounds. Which other clinical findings is likely to be present? Cheilosis and stomatitis B. Confusion, ophthalmoplegia, and ataxia C. Swollen gums, bruising, and hemarthrosis D Dermatitis and mental status change Show explanation 44% E Ptosis , anhydrosis, and miosis Show explanation 15%

Major takeaway Carcinoid syndrome is associated with seratonin secreting neuroendocrine cells. An overproduction of seratonin can lead to niacin deficiency and cause pellagra (diarrhea, dermatitis, and mental status changes). Main explanation The patient has carcinoid syndrome, a disease which features a neuroendocrine cell (NEC) tumor which secretes a large amount of serotonin in an episodic manner. Serotonin causes the patient's symptoms of flushing, wheezing, diarrhea, and hypotension. In addition, carcinoid syndrome patients often have right-sided heart valve defects, most commonly tricuspid regurgitation. The high amount of serotonin in systemic circulation causes fibrotic plaques to develop on the right side of the heart. Serotonin is broken down in the lung which protects the left side of the heart. Serotonin is a derivative of tryptophan. Because of the excessive amount of serotonin produced in carcinoid syndrome, there may be a deficiency of other products which are also tryptophan derivatives. Niacin is one of those products. Therefore, some patients with carcinoid syndrome may be characterized by niacin deficiency, also known as pellagra. Pellagra is defined by the classic triad of dermatitis, dementia, and diarrhea (The 3 D's of pellagra). Not A. Riboflavin is an essential vitamin used in the production of flavin adenine dinucleotide and flavin mononucleotide. Symptoms of riboflavin deficiency include stomatitis, cheilosis, anemia, and itchy eyes. Riboflavin deficiency is common in developing countries but rare in developed countries with riboflavin fortified grain and rice products Not B. Wernicke encephalopathy is caused by a deficiency in thiamine and is particularly common in alcoholics. Wernicke is defined by the classic triad of confusion, ophthalmoplegia, and ataxia. In patients suspected of Wernicke encephalopathy, it is crucial to administer thiamine before glucose is given. Not C. Scurvy is a disease caused by a deficiency in vitamin C. Symptoms include swollen and bleeding gums, easy bruising, hemarthrosis, and neuropathy. Failure to treat scurvy will eventually lead to death. Vitamin C is found in vegetables and fruits such as kale, oranges, and lemons. Not E. Horner syndrome is a constellation of symptoms as a result of a lesion to the sympathetic chain. Symptoms include ipsilateral ptosis, anhydrosis, and miosis. Etiologies include multiple sclerosis, central nervous system tumors, and Pancoast tumors.

A 57-year-old woman comes to the clinic because of persistent coughing productive of blood-tinged sputum. She has had night sweats and chills for the past week. She is a nurse working on the infectious disease unit of the hospital. Her temperature is 37.5°C (99.5°F), pulse is 82/min, respirations are 18/min, and blood pressure is 120/80 mm Hg. A blood sample is drawn for quantiferon testing. Chest x-ray shows right hilar lymphadenopathy. Which of the following is most likely present in the affected lymph nodes of the lung? A. Caseous necrosis B. Gangrenous necrosis C. Coagulative necrosis D. Liquefactive necrosis E. Fibrinoid necrosis

Major takeaway Caseous necrosis is a type of unprogrammed cell death resulting in a soft, white foci of dead tissue. It is characteristic in tuberculotic lymph nodes. Main explanation This patient's hemoptysis, constitutional symptoms, and healthcare occupation, as well as her radiological finding, suggest a possible tuberculosis (TB) infection. An infection with Mycobacterium tuberculosis leads to caseous necrosis in the lymph node foci. Necrosis is a form of cell injury that causes cell death in living tissue. In caseous necrosis, the dead tissue has a cheesy appearance ('caseus' is Latin for 'cheese') and looks soft and white upon macroscopic examination. Under microscopy with H&E staining, the area looks like an acellular, proteinaceous mass surrounded by granulomatous inflammation. In TB, confluent caseous necrosis leads to extensive lung destruction and cavitation. Other causes of caseous necrosis include fungal infection (e.g. histoplasmosis, cryptococcosis, coccidioidomycosis) and syphilis. Not B. Gangrenous necrosis occurs following ischemia of the lower limbs or in the gastrointestinal tract. It is considered a type coagulative necrosis (dry gangrene), or, if superimposed infection occurs, liquefactive necrosis (wet gangrene). Not C. Coagulative necrosis is characteristic of necrosis secondary to ischemia and hypoxia, e.g. following an infarction in the heart, kidney, or adrenal gland. The tissue architecture is maintained, but the dead tissue and denatured proteins are apparent as a gelatinous substance. Not D. Liquefactive necrosis is seen following hypoxic infarction in the brain. The brain is rich in digestive enzymes and lipids, facilitating autodigestion of cells. Bacterial and fungal infections can also cause liquefactive necrosis. Not E. Fibronid necrosis results from deposition of immune complex and leaked fibrin in the vessel walls, leading to vascular damage. It is seen in malignant hypertension, post streptococcal glomerulonephritis, hyperacute graft rejection, and certain autoimmune diseases.

A 16-year-old boy, who recently immigrated to the United States from Africa, comes to the clinic because he has had intermittent fevers and headache for the past few weeks. He is human immunodeficiency virus positive and has not been adhering to his antiretroviral regimen. On physical examination, he is afebrile with nuchal rigidity and sensitivity to light. One month ago his CD4 count was 20 cells/uL. A lumbar puncture shows an elevated opening pressure of 220mm H2O. Microscopy shows a spherical cells with a zone of clearance surrounding each one. Which of the following is the best treatment? A. Amphotercin-B followed by flucytosine, and flucoazole B. Fluconazole C. Rifampin and isoniazid D. Corticosteroids E. Extended-course oral tetracycline

Major takeaway Cryptococcus neoformans is a fungal infection that causes meningitis in immunocompromised hosts and is treated with amphotericin B. Treatment with amphotericin B may be followed by flucytosine, and is maintained with a regimen of fluconazole. Main explanation Cryptococcus neoformans is an encapsulated year and an obligate aerobe that can live in humans. A patient with human immunodeficiency virus with fever and nuchal rigidity should have Cryptococcus meningitis on the differential. Symptoms include fever, headache, irritability, dizziness, confusion, and possibly seizures. Of note, cryptococcal infection in HIV pediatric patients is actually quite rare in comparison to adults. The reason for the reduced incidence in children is not known. However, in this case, the *India Ink staining of the polysaccharide capsule is diagnostic of Cryptococcus*. The first line treatment is typically amphotericin B followed by flucytosine, and is maintained with a regimen of fluconazole.

A 67-year-old man comes to the emergency department after a fall at home walking to the kitchen from the living room. His daughter reports that he fell on his head, and later reported a very bad headache. The initially drowsy patient becomes comatose, and his breathing becomes apneic. His systolic blood pressure rises steeply. Which of the following findings would also be associated with Cushing triad? A. Bradycardia B. Decreased pulse pressure C. Hypothermia D. Mydriasis E. Myoclonus

Major takeaway Cushing triad is characterized by widened pulse pressures, respiratory irregularity, and bradycardia. These three findings suggest increased intracranial pressure, a medical emergency. Main explanation Cushing triad is a set of findings that are associated with an increase in intracranial pressure. The three signs are widening pulse pressure (rising systolic, declining diastolic), irregular respiration or apnea, and bradycardia. *Cushing triad is different to the Cushing reflex* (which is an increase in sympathetic outflow in response to cerebral ischemia, which initially results in raised blood pressure, due to raised peripheral vascular resistance and tachycardia). The late phase of the Cushing reflex results in the signs of Cushing triad. There can be many causes of increased intracranial pressure. Some of the common causes are brain tumors, subdural or epidural hematomas, traumatic injury, choroid plexus papilloma, and large cerebral infarctions. Regardless of the cause, increased intracranial pressure is a medical emergency and must be managed immediately by treating the underlying cause, and possibly removing intracranial fluid. Myoclonus refers to the involuntary twitching of muscles. There can be many causes of myoclonus including brain tumors, stroke, and drugs, but not Cushing triad

A 40-year-old male sheep farmer comes to the clinic with a lesion to his right forearm. The lesion began seven days ago as a small papule of about one centimeter in diameter. Physical exam shows a 3 centimeter lesion to the proximal right forearm with a black center with surrounding tissue swelling. A tissue sample of the lesion grows a gram positive bacillus. Which of the following toxins is the most likely to causing the patient's symptoms? A. Alpha toxin B. Toxic Shock Syndrome Toxin C. Lethal toxin D. Lethal toxin and edema factor E. Epidermolytic (Exfoliative) Toxin F. Adenylase toxin

Major takeaway Cutaneous anthrax is caused by the bacteria Bacillus anthracis, a gram-positive, spore-forming rod, and presents as a black eschar. This painless necrotic lesion is caused by the edema factor and lethal toxin released by B. anthracis. Main explanation This man is suffering from cutaneous anthrax, caused by Bacillus anthracis. B. anthracis is a gram-positive, spore-forming rod that is zoonotic in nature. The patient probably contracted it from being in contact with contaminated wool.* Farmers and shepherds* are at high risk of contracting this infection. This bacteria possesses edema factor, which increases cyclic AMP (cAMP) levels leading to vascular leakage and edema, as well as lethal toxin, which increases TNF-α and IL1-β promoting shock and death via hemorrhage. Both toxins are involved in the pathogenesis of cutaneous anthrax, which is characterized by the formation of eschars. *Eschars are large ulcerations with black, necrotic centers.* This patient should be treated aggressively and quickly to prevent severe complications. Antibiotic treatment should consist of *ciprofloxacillin or doxycycline* for 60 days. If the strain is susceptible, amoxicillin or amoxicillin-clauvunate may be used to complete the course as well.

A 75-year-old woman comes to the emergency department because of nausea and vomiting, abdominal pain, double vision and describes seeing green/yellow outlines around objects. She reports a history of NYHA Class III heart failure, atrial fibrillation, and bipolar disorder. Her current medication list is quite extensive and includes ramipril, bisoprolol, spironolactone, digoxin, amiodarone, and lithium. Which of her medications is most likely to be responsible for her symptoms? A. Amiodarone B. Digoxin C. Lithium D. Spironolactone E. Ramipril

Major takeaway Digoxin toxicity classically presents with nausea, vomiting, bradycardia, and visual symptoms include altered color perception and the appearance of green and/or yellow "halos." Main explanation Patients with digoxin toxicity often present initially with nausea, vomiting, loss of appetite, and bradycardia. Additional characteristic signs of digoxin toxicity include visual symptoms such as diplopia and visual blurriness, altered color perception, and the appearance of green or yellow "halos" surrounding objects. Digoxin toxicity can also manifest with various arrhythmias such as a prolonged PR interval, accelerated junctional rhythm and biventricular tachycardia. Several additional, rarer side effects of elevated digoxin levels include confusion, delirium and abdominal pain. Amiodarone, an antiarrhythmic drug commonly used for ventricular arrhythmias, increases the concentration of digoxin by reducing the gastrointestinal elimination and renal tubular secretion. *Dose reduction and careful monitoring of digoxin levels are required when using the two medications together.* Side effects of amiodarone include hypotension, peripheral neuropathies, tremor, photosensitivity, pulmonary fibrosis, and optic neuritis. A. Amiodarone greatly increases the bioavailability of digoxin in the body. Often when amiodarone is initiated in a patient currently taking digoxin, the dose of digoxin must be decreased by approximately 50% B. The presenting symptoms of lithium toxicity in patients with serum lithium levels of >1.5 mEq/L include coarse hand tremors, vomiting, persistent diarrhea, confusion and ataxia. Serum lithium levels >3 mEq/L results in CNS depression, seizures, and even coma. C. Spironolactone can result in side effects that include hyperkalemia and an increased serum creatinine. Gynecomastia and impotence are additional side effects that can be seen with spironolactone.

A 25-year-old woman comes to the emergency department because of abdominal pain. She describes it as intermittent dull, throbbing pain in the left lower quadrant. She complains of nausea and vomiting in the morning that usually goes away in the middle of the day. She denies the possibility of being pregnant as she uses oral contraceptive pills. Her past medical history includes treatment of a sexually transmitted infection with doxycycline one year ago, and a history of depression, currently being treated with paroxetine. Serum beta-hCG levels are 2225 mIU/mL. A transvaginal ultrasound scan (TVUS) is obtained and is negative for an intrauterine gestation. Which of the following features on history put this patient at an increased risk for the most likely diagnosis? A. Prior infection with Gardnerella vaginalis B. Prior infection with Chlamydia trachomatis C. Age D. Ethnicity E. Oral contraceptive pill use

Major takeaway Ectopic pregnancy is suspected when there is a positive b-hCG test and nothing is seen in the uterine cavity on transvaginal ultrasound. Pelvic inflammatory disease (PID) is a major risk factor for Fallopian tube damage and ectopic pregnancy. Main explanation This is a presentation of an ectopic pregnancy in a young woman. Characteristic symptoms of an ectopic pregnancy include lower abdominal pain, amenorrhea and vaginal bleeding. A transvaginal ultrasound and β-hCG levels are key diagnostic tests. Risk factors of ectopic pregnancy include prior pelvic inflammatory disease (PID), intrauterine device (IUD) use, prior ectopic pregnancy, tubal sterilization/reconstructive tubal surgery, and diethylstilbestrol (DES) exposure. The patient had a prior infection with Chlamydia trachomatis, which puts her at risk for developing PID. In her past history, it was stated that she was treated with doxycycline for a sexually transmitted disease (STD), which is the drug of choice for Chlamydia. An episode of PID will increase the risk of ectopic pregnancy by six-fold after one episode. The scarring of the Fallopian tubes due to the organism can prevent the proper travel of the zygote into the uterus allowing implantation in the Fallopian tube.

A 70-year-old woman comes to the emergency department because of an altered mental status. The patient was found by a neighbor unarousable on the couch, with multiple empty bottles of medications on the floor next to her. Social history shows alcohol abuse. Physical examination reveals an awake female with fluctuating level of consciousness, not oriented to time or place, with no focal deficits. Which of the following is most likely to be seen? A. Hallucination B. Insidious course C. Multi-infarct dementia D. Irreversibility E. Normal vital signs

Major takeaway Features of delirium include fluctuating level of consciousness; presence of hallucinations, disorientation, and abnormal vital signs. Delirium involves a waxing and waning course, and are cause by reversible entities such as infections, medications, postoperative periods, alcoholism, electrolyte imbalances, and other medical conditions. Main explanation The term organic brain syndrome is used to distinguish changes in cognitive/behavioral functions due to physical causes from those due to psychiatric causes. Organic brain syndrome can be divided into two major subgroups: acute (delirium or acute confusional state) and chronic (dementia). It is often difficult to distinguish the two, especially in an acute care setting. There are certain clinical features that can help to distinguish the two entities. Delirium is caused by infections, medications, postoperative periods, alcoholism, electrolyte imbalances, and other medical conditions such as stroke, heart disease, seizures, hepatic and renal disorders. Level of consciousness is usually altered and fluctuating. Hallucinations, disorientation, and abnormal vital signs are frequently present. The course is usually waxing and waning, and involves rapid deterioration and is almost always reversible. As opposed to dementia in which, the causes are Alzheimer disease, Pick disease, and multi-infarct dementia. In dementia, the level of consciousness is usually preserved. There are usually no hallucinations or abnormal vital signs. The course is usually insidious and progressive and symptoms are typically irreversible. This patient is exhibiting signs and symptoms of delirium, and therefore, hallucinations are the most likely to be present because all the other choices are features of dementia.

A 26-year-old man comes to the clinic because of dark urine for the past day. He has also had a fever, chills, and back pain. He recently returned to the United States after a 3-month stay in Africa where he was diagnosed with a Plasmodium vivax infection and was initiated on appropriate treatment with primaquine four days ago. Serum studies are shown below:Peripheral blood smear shows erythrocytes that are dark in color and somewhat deformed. A urine dipstick done in the office is positive for blood. Which of the following tests is most likely to identify the etiology of this man's pathology? Hbg 6.7 (NL 13.5 - 17.5) Leukocytes 12000 (NL 4,500 - 11,000) Platelets 360,000 (NL 150,000 - 400,000) Reticulocyte count 10% (NL 0.5-15%) A. MCHC B. Direct comb's test C. Heinz body preparation D. Hemoglobin electrophoresis E. Serum ferritin concentration

Major takeaway Glucose-6-phosphate dehydrogenase (G6PD) deficiency is associated with acute hemolytic anemia due to damage of erythrocytes following oxidative stress by certain medications. Peripheral blood smear shows Heinz bodies, precipitants found in erythrocytes as they are damaged and denatured. Main explanation This man's signs and symptoms of fever, chills, back pain, and dark urine, in association with exposure to primaquine is characteristic of glucose-6-phosphate dehydrogenase (G6PD) deficiency, which shows Heinz bodies on peripheral blood smear. G6PD deficiency is an inherited disorder of the G6PD enzyme which protects erythrocyte membranes from oxidative damage. This man's symptoms are associated with a state of acute hemolytic anemia as his erythrocytes sustain damage, provoked by the recent exposure to primaquine. In individuals with G6PD deficiency, oxidant stress can be induced by medications such as primaquine, dapsone, sulfa, fava beans, or by infection. The lack of G6PD results in the inability to neutralize the hydrogen peroxide by-product of erythrocyte metabolism which leads its accumulation and damage. As hemoglobin denatures, precipitants known as Heinz bodies are formed which can be identified on a peripheral smear. The confirmatory test for G6PD deficiency is the Heinz body preparation (diagnostic during periods of active hemolysis) which would be followed up with an enzyme assay for G6PD as a confirmatory study.

A 4-year-old boy is brought to the emergency department by his mother because of a 1 day history of rash. She says that he had Streptococcal pharyngitis several weeks ago, but was in his normal state of health until 5 days ago when he began complaining of right hip pain. This subsided and was replaced by right knee and ankle pain one day ago. At the same time, he developed a rash of dark red spots scattered on his legs, buttocks, and forearms. He also began complaining of nausea and stomach pains 2 hours ago. Physical examination shows a purpuric rash located in dependent areas, and a swollen right knee without effusion, erythema, or warmth. Urinalysis shows microscopic hematuria with proteinuria. Serum electrolytes, including creatinine, are within normal limits. Which of the following is the most likely diagnosis? A. Systemic lupus erythematosus B. Rheumatic fever C. IgA nephropathy D. Reactive arthritis E. Henoch-Schonlein purpura

Major takeaway Henoch-Schönlein purpura is characterized by the classic clinical triad of palpable purpura petechia in dependent areas, migratory arthritis, and abdominal pain. Main explanation This child is suffering from a classic case of Henoch-Schönlein purpura (HSP). The classic clinical triad of HSP is palpable purpura/petechia (in dependent areas, see image below), migratory arthritis, and abdominal pain. However, the arthritis and abdominal pain can often precede the rash. Other symptoms include microscopic hematuria and proteinuria (which is only rarely in the nephrotic range). Intussusception and gastrointestinal hemorrhage can also occur. HSP is an IgA-mediated vasculitis with IgA and C3 immune complexes as the offending agents. It is often associated with a prior Streptococcal infection, but can follow an infection of various etiologies, including viruses and other bacteria. Not A. Arthritis is a symptom of systemic lupus erythematosus(SLE). However, this patient has none of the other associated symptoms. 4 or more of the following must occur: Serositis , Oral ulcers, Arthritis, Photosensitivity, Blood cytopenias, Renal disease, Anti-nuclear antibody (ANA),Immunoglobulins, Neurologic symptoms, Malar rash, Discoid rash Not B. Arthritis may be a presenting symptom of acute rheumatic fever following Streptococcal infection that was not properly treated by antibiotics. However, other criteria must also be met, including J- "joints": migrating, oligoarticular arthritis O- actually a heart, referring to carditis and valvular damage N- subcutaneous nodules E- erythema marginatum S- Sydenham chorea Not C. About half of patients with IgA nephropathy have one or more episodes of macroscopic hematuria following an upper respiratory infection. Other patients may have microscopic hematuria or proteinuria. Not D. Reactive arthritis is an autoimmune phenomenon where antibodies to an infectious agent develop and have intrinsic cross-reactivity with self antigens. This leads to joint inflammation (usually oligoarthritis) which is sometimes migratory. It can be associated with other symptoms: conjunctivitis, urethritis, arthritis (can't see, can't pee, can't climb a tree)

A 27-year-old woman comes to the emergency department because of severe vomiting for the past two days. She is 9 weeks pregnant and began experiencing regular morning sickness in the form of mild nausea, with occasional episodes of vomiting, 7 weeks into her pregnancy. Her episodes of vomiting have since become more frequent. In the past 2 days, she says she has vomited over 25 times. Her other complaints include fatigue, dizziness, and epigastric pain associated with vomiting. Physical examination shows she is afebrile, pulse is 107/min, and blood pressure is 120/74 mmHg. Her arterial blood gas shows: pH 7.51pCO2 46 mmHgpO2 80Bicarbonate 32 Which of the following acid-base disturbances is present in this patient? A. Metabolic alkalosis B. Metabolic acidosis C. Respiratory alkalosis D. Respiratory acidosis

Major takeaway Hyperemesis gravidarum can result in a metabolic alkalosis. This is indicated by an elevated pH and an elevated serum bicarbonate level. This is caused by the *loss of hydrogen, chloride and potassium within gastric secretions* H+, Cl-, K+ Main explanation This patient has hyperemesis gravidarum. It is a complication of pregnancy which involves extreme nausea and vomiting and *can result in dehydration and malnutrition if not treated*. Onset usually occurs in the first trimester of pregnancy, with resolution at the beginning of the second trimester. Treatment is supportive with IV fluids, antiemetics, and nutritional support, if necessary. Metabolic alkalosis can occur in hyperemesis gravidarum due to the loss of ions in the gastric secretions. Gastric secretions are high in hydrogen (H+) and chloride (Cl-) in the form of hydrochloric acid and also high in potassium (K+). Therefore *this patient will develop a hypochloremic hypokalemic metabolic alkalosis*. The loss of hydrogen will initiate the alkalosis and the further loss of potassium and chloride will sustain the alkalosis because their loss favors the reabsorption of bicarbonate and impairs its secretion within the kidney.

A 58-year-old man comes to the emergency department because of renal insufficiency. He has nausea, weakness, diffuse abdominal pain, and a tingling sensation in his arms and legs. Physical examination shows areflexia. An ECG is obtained and is shown below. Which of the following is the most likely diagnosis? A. Hyperkalemia B. Hypernatremia C. Hypocalcemia D. Hypokalemia E. Hyponatremia

Major takeaway Hyperkalemia is an elevated concentration of potassium (K+) in the blood and diagnosis is made with the measurement of potassium, glucose, creatine kinase, and cortisol. ECG determines if there is a significant risk of arrhythmia. Main explanation Hyperkalemia refers to an elevated concentration of potassium (K+) in the blood. *Normal serum potassium levels are between 3.5 and 5.0* mEq/L. Levels above 5.0 mEq/L indicate hyperkalemia, and below 3.5 mEq/L indicate hypokalemia. Hyperkalemia is characterized by malaise, palpitations, and muscle weakness. Hyperventilation indicates a compensatory response to metabolic acidosis, which is one of the causes of hyperkalemia. Often, however, the problem is detected during screening blood tests for a medical disorder, or after hospitalization for complications such as cardiac arrhythmia or sudden cardiac death. Kidney disease and medication are common causes. For diagnosis, the measurement of potassium is repeated and blood tests for kidney function (creatinine and blood urea nitrogen), glucose, creatine kinase, and cortisol are needed. ECG is obtained to determine if there is a significant risk of arrhythmia. With mild to moderate hyperkalemia, there is prolongation of the PR interval and development of peaked T waves. Severe hyperkalemia results in a widening of the QRS complex, and the ECG complex can evolve to a sinusoidal shape. When arrhythmias occur, or when potassium levels exceed 6.5 mmol/l, emergency calcium gluconate and lowering potassium blood levels is the most appropriate management. A. Hypernatremia is an elevated sodium level in the blood (above 145 mEq/L) and is caused by a deficit of free water in the body. Hypernatremia is characterized by thirst, brain cell shrinkage (include confusion, muscle twitching or spasms), seizures, and coma. C. Hypocalcemia is the presence of low serum calcium levels in the blood (below 2.12 mmol/L) and causes are hypoparathyroidism, vitamin D deficiency, and chronic kidney disease. Symptoms of hypocalcemia are neuromuscular irritability, electrocardiographic changes, and seizures. D. Hypokalemia is the condition in which the concentration of potassium in the blood is below 3.5 meq/l. Symptomatology varies if it is mild, moderate, or severe hypokalemia. ECG findings are flattened or inverted T waves, a U wave, ST depression, a wide PR interval, and a prolonged QT interval. E. Hyponatremia is low sodium concentration in the blood (below 125 mEq/L) and is characterized by nausea, vomiting, headache, short-term memory loss, confusion, lethargy, fatigue, loss of appetite, irritability, muscle weakness, spasms or cramps, seizures, decreased consciousness, or coma.

A 45 year old man is brought to the emergency department by his family after an episode of syncope. Two days ago, he began experiencing nausea, diarrhea, and abdominal cramps. He lost consciousness upon getting out of bed, though quickly came back to baseline. He has a nearly 10-year history of dizziness after standing and easy fatiguing. His vital signs are significant for a fever to 101F, blood pressure of 90/60 while sitting, and tachycardia. His oral mucosa shows mildly darkened gums. Axillary sweat is present. Cardiac and pulmonary exams are normal, with a JVP estimated at 6 cm. His extremities are warm and well-perfused. His metabolic panel is as follows:Na: 130 mEq/L Cl: 105 mEq/L BUN: 28 mg/dLK: 6.1 mEq/L CO2: 15 mm Hg Cr: 1.15 (baseline 0.95)Glucose: 65The patient is started on intravenous fluids. Which of the following is the best next step in the treatment of this patient? A. Insulin with 1 ampule of D50W B. Broad spectrum antibiotics (Vancomycin + Piperacillin/tazobactam) C. Rehydration with 3% saline solution D. Stress dose IV hydrocortisone (100 mg bolus) E. Goal directed care with pressors (e.g. norepinephrine)

Major takeaway Hyponatremia can be seen in the setting of adrenal insufficiency. This can arise due to conditions such as Addison's disease. Treatment would initially involve high-dose corticosteroid replacement. Main explanation Adrenal insufficiency is an important component in the differential diagnosis of hyponatremia. Causes of adrenal insufficiency can include rapid cessation of exogenous corticosteroids and Addison's disease.This patient is suffering from an adrenal crisis - characterized by lethargy, hypotension, hyperkalemia, hyponatremia, syncope, and fever. An adrenal crisis is an acute exacerbation or onset of adrenal insufficiency. This patient's insufficiency is likely due to Addison's disease, an autoimmune adrenalitis characterized by hyperpigmentation (from overproduction of ACTH), easy fatiguing, and consistent orthostatic hypotension, among other symptoms. Other causes include invasion by cancer/fungus/tuberculosis, congenital abnormalities, exogenous corticosteroid withdrawal, and adrenal hemorrhage (Waterhouse-Friedrichsen syndrome). Treatment of an adrenal crisis of any etiology involves-high dose corticosteroid replacement.

A 9-year-old boy comes to the clinic because of a severe headache. He returned yesterday from a family vacation in Sardinia. He spent the days outside playing near a river with his cousins. His mother says that he gets occasional headaches that are relieved by taking a nap, but he has never experienced anything like this before. Physical examination shows that his neck is stiff and the presence of a rash on his precordium. Penlight examination of his eyes is significant for photophobia. He is also febrile and seems to have weakness in all four limbs. A blood test demonstrates pleocytosis. Which of the following is the most likely diagnosis? A. Giant cell arteritis B. Silicosis C. Stroke D. West Nile fever E. West Nile viral meningitis

Major takeaway If it suspected that a patient has West Nile fever but neurological symptoms are present, consider the diagnosis of West Nile viral meningitis. Main explanation This child has West Nile viral meningitis. Given that there is neurological disease present, it can not be diagnosed as West Nile fever (typified by absence of neurological symptoms). West Nile meningitis is difficult to distinguish from viral meningitis but is an important disease to consider especially if a traveller has returned from an area where mosquitos and West Nile virus are prevalent. West Nile virus is a flavivirus, in the same family as yellow fever, dengue, hepatitis c virus, and St. Louis encephalitis virus. As a group, these viruses cause devastation to many groups of peoples that live outside of industrialized areas. Early diagnosis and supportive care are imperative.

A 28-year-old G3P2 pregnant woman at 12 weeks gestation is brought to the psychiatric emergency department after purchasing 2 exotic sports cars as a surprise gift for her family. She hid them in different parts of town with the door unlocked and the key in the ignition. She presents with pressured speech, distractibility, and claims she has slept very little since Sunday. Which of the following is the appropriate management for this patient? A. Valproic acid B. Lithium C. Electroconvulsive therapy D. Carpamazepine E. Lamotrigine

Major takeaway In female patients with mania who are pregnant or at high risk of unintended pregnancies, it is best to avoid teratogenic drugs such as valproate, carbamazepine, and lithium (know the specific fetal malformations!). Electroconvulsive therapy is the treatment of choice in pregnant women. (But PAEA answer used lamictal too...) Main explanation This patient should be treated with electroconvulsive therapy. She is likely experiencing a manic episode, the symptoms of which can be remembered with the mnemonic "DIG FAST". - Distractibility - Indiscretion - increased indulgence in pleasurable activities - Grandiosity - Flight of ideas - rapid speech that flits from one idea to another with only superficial connections - Activity increase - Sleep defect - Talkativeness In order to be considered a true manic episode, these symptoms must last for a week and significantly affect the patient's life and ability to function. In female patients who are pregnant or at high risk of unintended pregnancies, it is best to avoid teratogenic drugs such as valproate, carbamazepine, and lithium. Valproic acid is severely contraindicated and can cause neural tube defects, carbamazepine can cause spina bifida and facial abnormalities, and lithium causes cardiac malformations such as Ebstein anomaly. A. Valproic acid is another first-line therapy for patients with bipolar disorder, but has an X rating for pregnancy. It will cause major congenital malformations including neural tube defects and mental retardation. C. Lithium is used to control manic episodes in patients with bipolar disorder. Lithium should be avoided in the first trimester of pregnancy. D. Carbamazepine can be used to treat manic or mixed episodes in patients with bipolar disorder, but should be not be given to pregnant women. It can cause spina bifida, craniofacial abnormalities, and hypospadias in the fetus. E. Lamotrigine has a class C rating, indicating that risk in pregnancy cannot be ruled out. Though it is effective in treating mania, there are better methods of treating this patient.

A 2-year-old girl is brought to the emergency department by her parents because they saw flecks of blood in her stool. Her mother says that the little girl doesn't seem to be having any abdominal pain and that she has been eating well. Her temperature is 37°C (98.6°F), pulse is 99/min, respirations are 16/min, and blood pressure is 100/70 mm Hg. Her abdominal examination shows no abnormalities. Which of the following is the most appropriate next step in management? A. Abdominal CT scan B. Xray of abdominal C. Abdominal exploration D. Mesenteric arteriography E. Technetium-99m pertechnetate scan

Major takeaway In hemodynamically stable patients with possible Meckel diverticulum, a Meckel scan (technetium-99m pertechnetate scan) should be performed. In patients with brisk bleeding, a mesenteric arteriography can be used to diagnose Meckel diverticulum. Main explanation Meckel diverticulum is the most common congenital anomaly of the GI tract. It is defined by the formation of a diverticulum of the small intestine from the vitelline duct, which is not completely obliterated. Gastrointestinal bleeding from a Meckel diverticulum is caused by an ulceration of the small bowel secondary to acid secreted by the ectopic gastric mucosa in the diverticulum. The bleeding is often painless. In hemodynamically stable patients, a Meckel scan (technetium-99m pertechnetate scan) should be performed. In patients with brisk bleeding, a mesenteric arteriography can be used to diagnose Meckel diverticulum. The classic description you should remember is the rule of twos: Meckel diverticulum occurs in about 2% of the population, is about 2 in long, is found about 2 ft from the ileocecal valve, approximately 2% of patients develop a complication over their lifetime, and the male to female ratio is 2:1.

A 20-year-old man is brought to the emergency department with high fever, headache, and confusion for one day's duration. He reports a skin rash that began earlier today. His temperature is 40°C (104°F); pulse is 120/min; respirations are 26/min, and blood pressure is 100/60 mmHg. Physical examination shows a toxic appearing male oriented only to himself with a positive Brudzinski sign. A petechial rash is seen on the trunk and upper extremities. Laboratory analysis shows normal electrolytes and renal function, a white blood cell count of 30,000 cells/uL, and a platelet count of 400,000/uL. His prothrombin time is 14 seconds. The patient's roomate and 8-year-old brother have been in contact with the patient since this morning. Which of the following antibiotics should be given to the brother and roommate as prophylaxis? A. Azithromycin B. Gentamycin C. Rifampin D. Ciprofloxacin E. Amoxicillin-clavunate

Major takeaway In meningococcal meningitis, treat all exposed persons prophylactically with rifampin. Other options include oral ciprofloxacin (contraindicated in children) and intramuscular ceftriaxone. Main explanation Neisseria meningitidis is a gram negative diplococci which causes meningococcal meningitis. Adolescents and young adults are most at risk for developing N. meningitidis meningitis. Meningococcal sepsis is life threatening and leads to petechiae and adrenal hemorrhage (also known as Waterhouse-Friderichsen syndrome). Patients with meningococcal septicemia should receive treatment with ceftriaxone. Because N. meningitidis is a contagious pathogen, prophylactic antibiotics should be given to all individuals in contact with the affected patient. For prophylaxis, the 3 options include rifampin, ceftriaxone, and ciprofloxacin. Ciprofloxacin is contraindicated in the pediatric population due to the risk of tendon rupture. Rifampin is the only medication listed that is appropriate and can be used safely in children and adults.

A 65-year-old man comes to the emergency department because of excessive salivation, sweating, and diarrhea for the past day. He was recently started on a new medication following months of difficulty swallowing, drooping eyelids, and blurry vision that worsened towards the evening. His temperature is 37°C (98.6°F), pulse is 55/min, respirations are 15/min, and blood pressure is 120/80 mm Hg. Which of the following medications is most appropriate to reverse this man's new-onset symptoms? A. Bethanechol B. Edrophonium C. Glycopyrrolate D. neostigmine

Major takeaway Initial therapy for myasthenia gravis with pyridostigmine, an acetylcholinesterase inhibitor can lead to an increase in acetylcholine resulting in excessive salivation, sweating, diarrhea, and bradycardia. Glycopyrrolate is an appropriate therapy for the reversal of these symptoms. Main explanation This man has symptoms of new-onset acetylcholine excess which is most likely caused by his recently-prescribed medication for myasthenia gravis. He requires treatment with glycopyrrolate, an antimuscarinic medication, to reverse his cholinergic symptoms which include excessive salivation, sweating, and diarrhea.This man's history of difficulty swallowing, drooping eyelids, and blurry vision are characteristic of myasthenia gravis which is an autoimmune disease which involves the body attacking the nicotinic acetylcholine receptors of the neuromuscular junction. Impulses are prevented from contracting the muscle, leading to weakness with repeated use. Initial medical therapy for myasthenia is with an anticholinesterase drug such as pyridostigmine, which this man is most likely taking. Pyridostigmine, edrophonium, and neostigmine are all cholinesterase inhibitors which increase acetylcholine at the neuromuscular junction and increase the strength of muscle contraction. An excess of acetylcholine may lead to increased salivation, sweating, diarrhea, and bradycardia. In serious cases, this may lead to cholinergic crisis which results in flaccid paralysis and respiratory distress. Not A. This man is most likely taking pyridostigmine for myasthenia gravis, causing an increase in acetylcholine resulting in excessive salivation and diarrhea. Bethanechol is a cholinergic direct agonist and would most likely make these symptoms worse. Not B. Edrophonium administration in this man would worsen, not reverse, his symptoms as it is an acetylcholinesterase inhibitor used in diagnosing myasthenia gravis. An increase in muscle strength with its administration is a positive result, but has been replaced with antibody testing due to a high false positive rate. Not D. Neostigmine would worsen this man's recent symptoms because it is an acetylcholinesterase inhibitor just like the pyridostigmine he is most likely taking for his recently diagnosed myasthenia gravis. His new symptoms are likely due to an excess of acetylcholine, and should be treated with antimuscarinic medication.

A 1-year old male infant is brought to the emergency department by his foster mother. She claims that the child has been acting strangely since this morning. His medical history is noncontributory. On physical examination, the baby looks lethargic and has alcohol and cigarette body odor. Skin examination reveals multiple bruises over shoulders, head, and buttocks. Lower limb exam shows a limited range of movement on his left leg. Fundoscopic exam shows bilateral retinal hemorrhages. His temperature is 36.7°C (98°F), pulse is 150/min, respirations are 50/min, blood pressure is 110/70 mmHg. Before finishing the medical exam, the foster mother wants to leave and asks how much longer is the visit going to take. When she is explained about the presumptive diagnosis, she gets angry and denies any form of treatment for the baby. Which of the following is the best immediate next step in management? A. Protect the child from the mother B. Request an urgent CT scan and call neurosurgery C. Report the mother to the police D. Obtain informed consent for surgery E. Contact child protective services

Major takeaway Physical abuse is the most common type of physical and sexual abuse in children. Victims that require an urgent medical intervention should be treated first. Once medical care has been established, then, protecting the child and reporting the abuse should be next. Main explanation The patient in the vignette is experiencing abusive head trauma (also known as "shaken baby syndrome"). Physical abuse is the most common type of child abuse. This term is defined as any intentional act causing injury or physical trauma to another person. Patients with physical abuse often present with soft tissue injuries, such as bruises, burns, and lacerations. Other findings may include broken bones, sexually transmitted diseases, and head trauma. Here, the patient is lethargic and has bilateral retinal hemorrhages. Both of these physical findings should be enough to raise *suspicion of subdural hematoma. Requesting an urgent CT scan* to confirm the diagnosis, in addition to, calling the neurosurgery team is the most appropriate conduct to follow since this condition is life-threating for the baby. Risk factors involved in physical abuse to children include prematurity, congenital anomalies, and physical disabilities. Caregivers are more likely to physical abuse of their children if they were abused during childhood. Physical abuse should be immediately reported to child protective services. However, when the victim of child abuse requires an essential medical intervention, then, care for the patient's needs is mandatory. Once proper care is established, protecting the child and reporting the abuse should be next. Not E. Contacting child protective services is also a management step, in this case, however, it is not the first step. Since the patient is most likely experiencing an acute *subdural hematoma*, treatment should be first directed towards treating his urgent condition. Protecting the child and contacting child welfare services comes second.

A 10-month-old male infant is brought to the emergency department because of intermittent crying and refusal to feed for the last six hours. He had one episode of vomiting, which was green in color but had no visible blood. He passed multiple loose, bloody stools on the way to the emergency room. Physical examination shows an irritable infant with a temperature of 39.2°C (102.5°F). Abdominal examination shows a palpable mass in the right upper quadrant. Which of the following is the most likely diagnosis?

Major takeaway Intussusception is the telescoping of one portion of the bowel into itself, most commonly occurring at the ileocecal junction. The classic triad of intussusception includes colicky abdominal pain, bilious vomiting, and red "currant jelly" stool. Diagnosis can be made with ultrasound or contrast enema. Main explanation Intussusception is the invagination or telescoping of one portion of the bowel into itself, usually the proximal portion drawn into the distal portion by peristalsis. This most commonly occurs at the ileocecal junction and compromises the blood supply to that portion of the intestine. It has a peak incidence in infants ages 5-12 months with a male predominance. Intussusception is usually idiopathic, often thought to be related to normal intestinal lymphoid hyperplasia. It can also be associated with recent enteric or respiratory viral infection. Anatomic lead points, such as Meckel's diverticulum, or pathologic lead points, such as a polyp or lymphoma, may be responsible.The classic triad of intussusception includes colicky abdominal pain, bilious vomiting, and red "currant jelly"stool; however, this is only present in 50% of cases. Neurologic signs, such as lethargy, seizure, and apnea, may also be seen and can delay diagnosis. Physical examination may reveal a sausage-shaped mass in the right upper quadrant. Ultrasound is often the first step toward diagnosis, revealing a 3-5 cm mass with a characteristic "target sign" appearance. Historically, contrast enema was the diagnostic procedure of choice as it was both diagnostic and therapeutic, reducing the need for surgical intervention.

A 38-year-old male is undergoing an umbilical hernia repair. The anesthetic plan is general anesthesia with endotracheal intubation. During the pre-operative assessment you direct your physical examination towards finding out if the patient will be a difficult intubation. Which of the following physical exam findings would be suggestive of difficult intubation? A. Mouth opening of 2 cm B. Ability for the patient to bite their upper lip with their lower jaw C. A neck circumference of 25 inches D. A thyromental distance of 7cm

Major takeaway Know the parameters for a uncomplicated airway and what they imply about the patient's airway. YOU WILL BE PIMPED ON THIS! Main explanation Determining if the patient is going to be a difficult intubation is a key aspect of the pre-operative assessment. Several factors indicate this: -A mouth opening of less than 3 cm. A small mouth will impair the insertion of a laryngoscope and the visualization of the vocal cords. -The inability of the patient to bite their upper lip with their lower jaw. This shows that their is limited anterior motion of the mandible at the temporomandibular joint. -A thyromental distance of less than 6 cm (three finger widths) is a good predictor of an airway that is displaced more anteriorly and therefore will be difficult to visualize. -A neck circumference of greater than 27 inches is associated with difficulty visualizing the glottic opening.

A 56-year-old male with history of hypertension and hyperlipidemia comes to the emergency department because of severe pain in both of his feet. He had an appendectomy at age 43 and a recent coronary angioplasty. His vital signs are within normal limits, and peripheral pulses are intact. The affected skin has a mottled appearance, with patterns of blue streaks that extend to just above the ankles. Both the dorsum and sole of the feet are involved. There is an ulcer present on the sole of the left foot. The mottled skin blanches upon pressure. Which of the following is the most likely diagnosis? A. Brucellosis B. Cholesterol emboli C. Fat emboli D. Henoch-Scholein purpura E. Microscopic polyangitis

Major takeaway Livedo reticularis in an extremity is a sign of small artery occlusion. During cardiac catheterization, cholesterol emboli can be dislodged from the aorta and will travel downstream before becoming stuck in extremity arterioles, causing low-level ischemia. Main explanation Cholesterol crystal emboli (atheroemboli) consist of pieces that are dislodged from a larger atherosclerotic plaque usually originating from the aorta. These emboli often lodge in small arteries, causing ischemia of the local tissue or organ. *A mottled pattern on the skin (livedo reticularis) is the most common cutaneous manifestation of a cholesterol embolus*, although it is not very specific for atheroembolism. In patients with cholesterol embolism, the livedo reticularis is usually bilateral. The feet and lower legs are often involved, but mottling can spread up to the thighs, buttocks, and back. Upper extremities are usually spared. During intravascular procedures, atherosclerotic plaques fragments can be dislodged and embolize. Peripheral pulses are usually maintained in cholesterol embolism.It is associated with many hypercoagulable states including polycythemia vera, multiple myeloma, antiphospholipid syndrome, cryoglobulinemia, and others. Not A. Brucellosis is an infection caused by the bacterium Brucella melitensisor brucella abortus. Dermatologic manifestations of brucellosis include macular, scarlatiniform, or papulonodular lesions and ulcerations that may resemble the patterns seen in livedo reticularis. Brucellosis is also associated with arthralgias, low back pain, sweating, and a moldy odor. Not C. Fat embolism usually occurs with the fracture of the pelvis or long bones such as the femur. It is characterized by a triad of features: hypoxemia, neurologic abnormalities, and petechial rash. Not E. Microscopic polyangiitis is characterized by constitutional symptoms like weight loss and fever along with purpura and myalgias. Cutaneous features of the lower limbs such as purpura or livedo reticularis may be present.

A 75-year-old man is brought to the emergency department because of a productive cough and a fever of 39.4°C (103°F) for the past 3 days. The man has dementia and is brought to the hospital from a nursing home. A sputum gram stain shows lancet shaped gram positive diplococci. A chest radiograph shows the following: Which of the following is the most likely diagnosis? A.Sarcoidosis B. Interstitial Pneumonia C. Aspiration pneumonia D. Small cell lung carcinoma E. Lobar Pneumonia

Major takeaway Lobar pneumonias show up as localized consolidations on chest radiographs. Streptococcus pneumoniae is a gram positive diplococci and is the most common cause of lobar pneumonia. Main explanation Lobar pneumonias are characterized by a consolidation of an entire lobe of the lung which can be easily seen on chest radiographs. Most cases of lobar pneumonia are bacterial in nature. The most common cause of lobar pneumonia is Streptococcus pneumonia (which occurs in 95% of all cases). S. pneumonia is a gram positive and alpha hemolytic diplococci. Gross pathologic examination of lobar pneumonia include congestion, red hepatization (secondary to exudates, neutrophils, and hemorrhage filling the alveolar air spaces), and gray hepatization (due to degradation of red cells within the exudate). Treatment for lobar pneumonia caused by S. pneumonia include *intravenous ceftriaxone, a third generation cephalosporin.* A. Sarcoidosis is an auto-immune disorder commonly affecting African American women. Sarcoidosis is associated with elevated levels of ACE, hilar lymphadenopathy, and respiratory distress. B. Interstitial pneumonia is often characterized by diffuse interstitial infiltrates from fibrosis formation or infection. Interstitial lung disease can be idiopathic but can also arise from environmental exposures such as silicon and asbestos. C. Aspiration pneumonia occurs when a patient aspirates pharyngeal contents into the lungs. Often, this will occur in the setting of altered mental status and unconsciousness. Aspiration pneumonia generally arises from anaerobic bacteria. D. Small cell carcinoma of the lung is a malignancy commonly associated with smoking and often metastasizes at time of diagnosis to the liver, adrenals, and brain. Because the tumor is extremely invasive and carries a poor prognosis.

A 70-year-old man comes to the office because of fatigue and pain in his jaw and ribs for the past 2 weeks. He has no history of trauma, but he says he is constipated, and a relative says that he has been more confused lately. He has a history of ischemic heart disease and osteoarthritis. Physical examination is non-contributory. Laboratory studies show normal anion-gap acidosis. Peripheral blood smear is obtained and is shown below. Chest X-ray shows lytic bone lesions of the ribs. Which of the following is the most likely diagnosis? A. Monoclonal gammopathy of undetermined significance B. Multiple myeloma C. Osteosarcoma D. Primary amyloidosis E. Waldenstrom macroglobulinemia

Major takeaway Multiple Myeloma symptoms can be remembered by the mnemonic CRAB:C: hyperCalcemia (causing constipation, stones, groans and psychiatric overtones) R: renal impairment A: anemia B: bones (pain, lytic lesions, and fractures) Main explanation Multiple myeloma (MM) is caused by a neoplastic proliferation of a single plasma cell line. Copious amounts of Ig, usually IgG or IgA are produced. It is characterized by bone pain due to osteolytic lesions (jaw and ribs), fractures, and even vertebral collapse. In addition, patients also suffer from anemia, pancytopenia (infections), renal failure (myeloma nephrosis yielding casts of Bence Jones) and hypercalcemia from bone (causing renal decompensation), and amyloidosis (amyloid protein is produced from bone marrow) For diagnosis: at least 10% abnormal plasma cells in bone marrow plus M-protein in serum OR M-protein in urine OR lytic bone lesions predominantly in the skull and axial skeleton. Other diagnostic clues include monoclonal proteins on serum and urine protein electrophoresis, urine monoclonal protein, lytic bone lesions on plain radiograph, 10% plasma cells on bone marrow biopsy, hypercalcemia, increased total serum protein, *peripheral smear (erythrocytes in rouleux formation) with pancytopenia, and urine with Bence Jones proteins*. Not A. This is usually an incidental finding that has no clinical presentation (asymptomatic). This is a monoclonal expansion of plasma cells with an M-spike that is a precursor to multiple myeloma. Not C. This is the second most common primary malignant tumor of bone (after multiple myeloma). It is most likely to occur in males 10-20-years-old, is commonly found in in the metaphysis of long bones, and is often in the distal femur, proximal tibial region (knee). Not D. This is the systemic deposition of amyloid, which is derived from Ig light chain. It can be associated with multiple myeloma. However, this is at a later stage when the plasma cell disorder is considered a systemic infiltrative pathology consisting of neuropathy, hepatomegaly, easy bruising, heart failure, arrhythmia, nephrotic syndrome. Not E. Waldenstrom macroglobulinemia is characterized by increased IgM and hyperviscosity. It is a type of cancer affecting B cells, a type of white blood cell. It is classified by an "indolent lymphoma" (ie: one that tends to grow and spread slowly), and is a type of lymphoproliferative disease, which shares clinical characteristics with the indolent non-Hodgkin lymphomas.

A 41-year-old woman, gravida 3, para 2, at 32 weeks gestation comes to the emergency department because of the onset of brisk, painless vaginal bleeding. She works as an actress and admits to exercising strenuously in order to maintain her physique, but not more than during either of her previous pregnancies. Physical examination shows that the uterus is non-tender and 32 cm above the pubic symphysis. Pelvic examination reveals the presence of a large amount of bright red vaginal blood that seems to be increasing its velocity of accumulation. The patient's blood pressure was 132/86 when she arrived at the hospital but is now 110/68. She appears pale and is becoming tachypneic. Which of the following is the most likely diagnosis? A. Chorioamnionitis B. Placenta previa C. Placental abruption D. Threatened abortion E. Vasa previa

Major takeaway Placenta previa should be suspected in all pregnant patients who present with painless vaginal bleeding during the third trimester. Imaging, usually via ultrasound, must take place before pelvic examination, which can worsen the situation if the placenta is in a dangerous position over the cervical os. The classic clinical presentation of placenta previa is painless, bright red vaginal bleeding. This diagnosis must be considered in all patients beyond 24 weeks' gestation who present with bleeding. Bleeding may be provoked with intercourse. Typically, the abdomen is soft and non-tender. Risk factors include previous C-section, high parity or multiple gestations, older maternal age, multiple gestation and chronic hypertension. Not C. Although placental abruption is an important cause of third trimester bleeding, it typically is accompanied by significant pain as the placenta abruptly detaches from the uterine wall. Not E. Vasa previa is when the fetal vessels, in contrast to the placenta itself in placenta previa, are in close proximity to the cervical os and may rupture. Clearly, this can result in an emergency situation for the fetus and often requires an immediate cesarean section delivery.

A 40-year-old man comes to the emergency department because of fever and diarrhea for the past 2 days. He has also had severe fatigue for the past 10 days. Past medical history is significant for human immunodeficiency virus diagnosed 3 years ago. Physical examination shows cervical lymphadenopathy and a palpable liver 3cm below the right costal margin. The CD4 count is <50/mm3. Which of the following is the most likely diagnosis? A. Cytomegalovirus B. Kaposi sarcoma C. Lymphoma D. Mycobacterium avium complex E. Pneumocystis jiroveci

Major takeaway Mycobacterium avium complex (MAC) should be considered in HIV positive patients with CD4 counts <50 presenting with cough, fever, right upper quadrant abdominal pain, fatigue, and weight loss. Main explanation A normal CD4 T-cell count is 1200/mm. Lymphadenopathy and minor infections (hairy leukoplakia, oral candidiasis) typically occur at <500/mm3. The immune system is severelycompromised and the patient progresses to AIDS after the cell count drops <200/mm3. AIDS can be diagnosed prior to this low of a CD4 count if the patient develops an AIDS defining opportunistic infection. Examples include: Kaposi sarcoma which can occur in patients with <500/mm3 Pneumocystis jirovecci pneumonia (PCP), histoplasmosis, and coccidioidomycosis which can occur in patients with <200/mm3 Toxoplasmosis, crytpococcosis, cytomegalovirus, and cryptosporidiosis which can occur in patients with <100/mm3 Mycobacterium avium complex (MAC) which can occur in patients with <50/mm3 Myobacterium avium complex presents similarly to tuberculosis. Symptoms includes cough, fever, abdominal pain (most commonly localized to the right upper quadrant), fatigue, and weight loss. Physical examination findings include lymphadenopathy and hepatosplenomegaly. Treatment of choice is with clarithromycin and ethambutol with or without the addition of rifabutin. Prophylaxis against MAC occurs at a CD4 count <50/mm3 with azithromycin or clarithromycin.

A 65-year-old man comes to the office because of dull chest pain, dyspnea, and heart palpitations. He occasionally feels his food "balling up" in his throat, causing significant discomfort in his chest. He denies exacerbation of his chest discomfort during exercise. He has tried taking calcium carbonate tablets to ease his symptoms with some success. His chest X-ray is shown below. Which of the following is the most appropriate surgical management of the patient's underlying condition? A. Gastropexy B. Heller myotomy C. Ivor-Lewis transthoracic esophagectomy D. Nissen fundoplication E. Ometoplasty

Major takeaway Nissen fundoplication is a surgical procedure to repair hiatus hernia. Hiatus hernias are characterized on chest radiographs by a mass in the posterior mediastinum containing air or an air-fluid level. Main explanation Nissen fundoplication is a common surgical treatment of hiatus hernia, a condition in which the proximal stomach protrudes superiorly through the diaphragm into the thoracic cavity. Clinical features of hiatus hernia include gastroesophageal reflux (GERD), epigastric or substernal pain, postprandial fullness, nausea, and retching. Chest radiographs show a mass in the posterior mediastinum that can be either filled with air or display a fluid-air level. In fundoplication, the gastric fundus is wrapped, or plicated, around the distal esophagus to strengthen the juncture between the esophagus and the diaphragm, thereby reducing the likely that abdominal viscera can pass through the diaphragm into the thoracic cavity. Often done laparoscopically, fundoplication is a relatively safe procedure with a quick recovery. A. A treatment for gastroesophageal reflux disease, gastropexy is a surgical procedure in which the stomach is sutured to the abdominal wall to reduce the likelihood of hiatus hernia. This patient has already herniated, so a Nissen fundoplication is necessary instead. B. Heller myotomy is a surgical procedure in which the esophagus is opened distally by cutting the lower esophageal sphincter to allow foods and liquids to pass into the stomach. It is a surgical treatment for achalasia. C. Ivor-Lewis transthoracic esophagectomy is used to resect cancers in the lower third of the esophagus. It combines a laparotomy with a right thoracotomy and an intrathoracic esophagogastric anastomosis E. Omentopexy is a surgical procedure to increase circulation to various abdominal viscera or cover injury sites. By attaching the omentum to different organs, holes may be patched (i.e. a perforated peptic ulcer, cysts from hyatid disease of the liver) or circulation increased by neovascularization (colorectal anastomosis, peripheral vascular disease).

A 75-year-old man comes to the emergency department because of flank pain. The pain began 8 hours ago and has persisted since. He denies nausea, vomiting, fever, or dysuria. His past medical history includes two previous kidney stones. Abdominal computed tomography scan confirms a 3 mm renal calculi at the ureterovesical junction. Which of the following is the most appropriate next step in management? A. Hydration B. Lithotripsy C. Renal angiography D. Suprapubic bladder catheter placement E. Surgical removal

Major takeaway Observation with conservative therapy, such as hydration, should be exercised prior to any more invasive procedures in small kidney stones. A 3 mm stone has a 70% chance of passing whereas a 7 mm stone only has a 5% probability of passing on its own. Main explanation This patient has a small stone <5 mm in size found on computed tomography scan. Stones of this size are very likely to pass on their own and should be given a trial of conservative management consisting of hydration and analgesia. Medium sized stones between 5 mm and 2 cm are best treated with non-invasive intervention such as shock-wave lithotripsy. Large sized stones >2 cm are often treated with invasive surgical intervention. Acute management of urinary obstruction can vary depending on the level of obstruction. Lower urinary tract obstruction can be acutely treated with a Foley or suprapubic catheter. In upper urinary tract obstruction, a nephrostomy tube can be employed to decompress the kidney and buy more time for the intervention. This patient has upper urinary tract obstruction but his mild clinical picture does not necessitate acute management. Not C. Renal angiography can detect abnormalities of the blood vasculature of the kidney making it useful in detecting *renal artery stenosis*. It is not useful in diagnosing urinary obstruction from nephrolithiasis Not D. A suprapubic bladder catheter is used to drain the bladder before an obstruction after the bladder has been definitively treated. However, this patient's obstruction is more proximal than the bladder. Not E. Surgery is indicated for nephrolithiasis that is blocking urine behind the stone such that it causes *hydronephrosis*. This patient's nephrolithiasis should be managed conservatively.

A 12-year-old girl is brought to the primary care office by her father because of swelling, pain, and erythema of her left cheek just in front of her ear and the left side of her jaw for the past week, causing her difficulty when eating. The patient's medical history shows that she has not had her routine booster immunizations. Physical examination shows that she has a fever of 38.9°C (102°F) and her mouth is extremely dry with no purulence noted. Which of the following is the most likely diagnosis? A. Acute viral parotitis B. Mastoiditis C. Salivary gland neoplasm D. Sjodren syndrome E. Submandibular sialadenitis

Major takeaway Parotitis refers to inflamed parotid glands which are exocrine glands that produce saliva. When inflamed, these glands become swollen and painful, and the superficial skin becomes erythematous. Parotitis can present unilaterally or bilaterally. It has various etiologies including bacterial and viral causes, and commonly the mumps virus. Main explanation This patient is experiencing parotitis, or inflammation of the parotid glands. These glands are within the mouth on both sides, sitting deep and just anterior to the ears. They are responsible for saliva production, and thus when they cannot function properly they result in a dry mouth. Parotitis has various etiologies, and the most common in children is the mumps virus, which is the most likely cause of this patient's symptoms. Mumps mostly affects children and causes parotitis among other viral symptoms. It typically affects the parotid glands but can also affect the submandibular glands as well. There is a vaccine to prevent mumps, and since this patient has not had updated immunizations, her symptoms should give a clue towards this diagnosis. Inflamed parotid glands are often presented in mumps along with systemic symptoms such as a fever and malaise. B. Mastoiditis is an infection of the mastoid bone resulting in inflammation of the mastoid air cells. Symptoms of mastoiditis include swelling, pain, and redness localized to the pre-auricular area. Fever and hearing loss are also common in acute conditions. D. Sjogren syndrome is an inflammatory disorder affecting exocrine organs such as the parotid glands ; thus, symptoms may include enlarged/swollen parotid glands. However, other exocrine organs would also be affected such as the other salivary glands including the submandibular and sublingual glands. Consequently, dry eyes and mouth are often also presented. C. Submandibular sialadenitis refers to an infection of the submandibular gland. This condition presents with symptoms similar to those here, i.e. swelling/pain of the affected area, fever. However, because this condition affects the submandibular gland, the symptoms would be focused on just the jaw. Pus is often present in the mouth.

A 67-year-old man is brought to the emergency department because of a skin infection. He says that he was bitten by a cat four days ago on his left forearm and that he feels pain, fatigue, and generalized body aches for the past day. Physical examination shows a 1 cm laceration on the left forearm with surrounding erythema, swelling, and increased warmth. Which of the following pathogens is the most likely cause of this patient's condition? A. Eikenella corrodens B. Pasteurella multocida C. Pseudomonas aeruginosa D. Staphylococcus aureus E. Bartonella henselae

Major takeaway Pasteurella multocida is the most common pathogen causing cellulitis related to cat or dog bites. It is a gram-negative bacterium that responds well to penicillin antibiotics. Main explanation This patient has cellulitis from a cat bite. Cellulitis refers to the inflammation of the skin and subcutaneous tissues which does not extend into deeper tissues, such as muscle or fascia. It often occurs following a break in the skin and is characterized by systemic symptoms, such as fever in severe cases. Pasteurella multocida is the most common cause of cellulitis following the bite of a cat or dog. Pasteurella multocida is a Gram-negative, nonmotile, penicillin-sensitive coccobacillus belonging to the Pasteurellaceae family. Strains belonging to the species are currently classified into five serogroups (A, B, D, E, F) based on capsular composition and 16 somatic serovars (1-16). P. multocida is the cause of a range of diseases in mammals and birds, including fowl cholera in poultry, atrophic rhinitis in pigs, and bovine hemorrhagic septicemia in cattle and buffalo. Many mammals (including domestic cats and dogs) and birds harbor it as part of their normal respiratory microbiota. Treatment includes local wound care, tetanus prophylaxis, and antibiotic therapy (e.g. amoxicillin/clavulanic acid). If left untreated a rapidly progressing cellulitis, septicemia, and serious illness can occur. Not B. Eikenella corrodens or the "fight bite" is a common cause of cellulitis following the bite of a human. This infection occurs on the knuckles after punching someone in the mouth. E. corrodens is a member of the HACEK group of organisms and can also cause endocarditis . Not E. Bartonella henselae is the primary pathogen responsible for cat scratch disease . This condition is characterized by solitary or regional lymphadenopathy that develops 1-2 weeks after a bite or scratch from a cat. There may also be a pustule at the site of injury

A 20-year-old woman comes to the emergency department because she thinks she may have been sexually assaulted. She says that she attended a party last night with a new group of friends and drank alcohol for the first time. She only remembers parts of the night and woke up in a friend's bed with only her shirt on. She put on the remainder of her clothing and came directly to the hospital without showering or brushing her teeth. She has been sexually active with two male partners in the past year, and has used barrier protection for intercourse every time. Pelvic examination does not show any bruising or tearing of tissues, and samples of vaginal fluid and pubic hair are collected for analysis. Which of the following is the most useful initial method for detecting semen in this patient? A. Culture for sexually transmitted diseases B. Gram stain C. Nitrazine paper test D. Polymerase chain reaction E. Wood lamp (UV-A light) exam

Major takeaway Patients who present for medical care after rape or sexual assault should be offered a forensic medical examination by a trained provider, and a Wood lamp, also called a blacklight, is a sensitive tool for the initial detection of semen or other foreign fluids. Main explanation Wood lamp (UV light), also called blacklight, will show fluorescence in the presence of alkaline phosphatase in semen. However, many other materials fluoresce such as urine or surgical lubricating jelly and thus it is a sensitive but not specific method for detecting semen. The sexual assault forensic evidence (SAFE) kit often recommends the use of a Wood's lamp. As this patient has not showered and is still wearing the same clothes that she had on the night before, the blacklight is a great tool for initial detection of semen or any foreign fluid in the vaginal, anal, and oral regions. In situations where signs of violent abuse, such as bruising and bleeding, are absent, the Wood lamp can be used to check the patient's entire body for samples to be analyzed.

A 65-year-old male comes to the emergency department because of chest pain and difficulty breathing. History reveals the patient was hit by a car and sustained a diffuse trauma to his chest. Past medical history reveals multiple hospitalizations for alcohol-related injuries and seizures. The patient is intubated in the emergency department for a flail chest segment and spends one week sedated in the intensive care unit. After a successful extubation, he is transferred to the floor where it is noted that he appears to be confused, has unsteady gait, and is easily agitated. Physical examination reveals asymmetric eye movements, most notably a lateral rectus palsy. The patient is oriented to himself and is unable to walk without assistance. Which of the following is the most appropriate treatment in this patient? A. immediate dose haloperidol injection B. Give high-dose cobalamin immediately C. give this patient folate IV thiamine D. STAT lorazepam

Major takeaway Patients with alcohol use disorder are at risk of developing Wernicke-Korsakoff syndrome. Treat this syndrome by administering intravenous thiamine to all patients at risk or showing signs of this syndrome. Main explanation Patients with alcohol use disorder would benefit from repletion of all of the vitamins and nutrients listed as answer choices. However, to treat Wernicke encephalopathy, intravenous thiamine is indicated. Wernicke encephalopathy is classically characterized as encephalopathy with confusion, gait ataxia, and ophthalmoplegia. If not treated with thiamine, it can progress to anterograde/retrograde amnesia, nystagmus, and confabulations known as Korsakoff psychosis. Korsakoff psychosis is irreversible. Classically this entire syndrome is worsened by giving a patient who with thiamine deficient a large bolus of intravenous glucose, causing acute swelling and death of neurons. Therefore, give all chronic alcoholics at risk of this syndrome a dose of thiamine prior to glucose. Not D: The mainstay of treatment for alcohol withdrawal is with benzodiazepines. Benzodiazepines, barbiturates, and ethanol are all active on GABA receptors, therefore withdrawal from one can be treated by the other. This patient is not showing any signs of alcohol withdrawal syndrome, such as autonomic hyperactivity, seizures, or visual hallucinations.

A 6-year-old boy from Pakistan comes to the emergency department with his mother because of a sore throat, headache, fever, malaise, and fatigue for the past five days. He says that he has a headache today and has vomited twice. His vomitus did not contain any blood. His temperature is 39.2°C (102.5°F), pulse is 114/min, and blood pressure is 132/83 mmHg. Physical examination shows neck stiffness, severe back pain, and weakness in his left leg. The tone and reflexes in his left leg are reduced. The sensory examination is normal. PCR of the cerebrospinal fluid shows the presence of an enterovirus. Which of the following is the most likely cause of his symptoms? A. Coxsackie virus B. Dengue virus C. Echovirus D. Polio virus E. Rhinovirus

Major takeaway Poliomyelitis is caused by the poliovirus (a type of enterovirus, transmitted fecal-orally). Infective symptoms present initially, and later preferential destruction of motor neurons causes paralysis. Main explanation Poliomyelitis is caused by poliovirus which is transmitted by fecal-hand-oral contamination. Initial symptoms include fever, headache, vomiting, diarrhea, neck stiffness, pain in the arms and legs. In around 1% of infections, poliovirus spreads along certain nerve fiber pathways, preferentially replicating in and destroying motor neurons within the spinal cord, brainstem, or motor cortex. This leads to the development of paralytic poliomyelitis, the various forms of which (spinal, bulbar, and bulbospinal) vary only with the amount of neuronal damage and inflammation that occurs, and the region of the CNS affected. Most *often it affects young children and may result in infantile paralysis*. Most infections are contained as short-term, mild viremia. If the viremia persists, viruses can be carried to the central nervous system through its blood supply. The virus then spreads along specific pathways in the spinal cord and brain. The virus infiltrates the motor neurons of the anterior horn of the spinal cord. The invasion of motor neurons causes various degrees of flaccid paralysis over a period of a few hours to several days.

A 68-year-old man comes to the emergency department because of suprapubic pain and decreased urination for the past 24 hours. He was commenced on benztropine one week ago, as he has been experiencing extrapyramidal side effects from his antipsychotic medication. His past history is significant for hypertension, benign prostatic hyperplasia, and chronic schizophrenia. His vital signs show no abnormalities. Examination shows suprapubic tenderness. An ultrasound scan of the bladder shows a post-void residual volume of 800 mL. Blood electrolytes show an elevated creatinine (1.8 mg/dL). Which of the following is the most likely diagnosis? A. Intrinsic renal vascular disease B. Post-renal acute kidney injury C. Intrinsic acute kidney injury D. Renal papillary necrosis E. Pre-renal kidney injury

Major takeaway Post-renal acute kidney injury is caused by urinary tract obstruction, this most often occurs in men with benign prostatic hyperplasia. Anti-cholinergic medication can cause acute urinary retention in this population. Main explanation Benign prostatic hyperplasia (BPH) is a common disorder in ageing men. BPH is characterised by lower urinary tract symptoms such as increased urinary frequency, nocturia, urgency, slow urinary stream and incomplete emptying. These symptoms develop slowly and progress gradually over a period of years. Benztropine is an anticholinergic medication used to treat extra-pyramidal side effects, *it is an inappropriate medication for patients with BPH. Anticholinergics can worsen BPH, leading to urinary retention*. This has occurred in this patients case, resulting in a secondary acute kidney injury (AKI). AKI is characterized by an abrupt but often reversible decline in glomerular filtration rate leading to an elevated BUN and creatinine. In this case the cause of the AKI is the obstructed urinary tract, and therefore causes postrenal azotemia. The initial management of acute urinary retention is prompt bladder decompression by catheterization. Men with a history of BPH should be referred for evaluation by a urologist, and should be started on an alpha-1-adrenergic antagonist, most commonly tamulosin is used. Tamsulosin works as an alpha-blocker to relax the muscles of urination and the prostate itself to allow proper voiding. C. Intrinisic acute kidney injury is due to damage to the glomeruli, renal tubules, or interstitium. Common causes include glomerulonephritis, acute tubular necrosis, and acute interstitial nephritis. D. Renal papillary necrosis is characterised by sloughing of the renal papilla. This would present with hematuria, proteinuria, and flank pain, which is not seen in this patient. E. Pre-renal acute kidney injury is caused by a reduced glomerular filtration rate in hypovolemic states, such as acute hemorrhage, congestive heart failure, or unreplenished fluid loss. There is no evidence of renal hypoperfusion in this patient.

A 70-year-old Asian-American woman comes to the urgent care office after developing blurred vision, headache, and excruciating right eye pain upon exiting a movie theater. Recent medications include a large dose of oral pseudoephedrine and used intranasal phenylephrine spray for seasonal allergies. She has worn corrective lenses for hyperopia for many years, and has no other known medical issues. Ophthalmic examination shows enlarged right conjunctival vessels and an opacified right cornea. Her left eye shows no abnormalities. She has difficulty counting fingers with her left eye covered but not with her right eye covered. Her right eye is poorly reactive to light. Which of the following diagnostic tests would best confirm the most likely diagnosis? A. Applanation tonometry B. CMP and CBC C. Dilated-pupil fundus exam D. MRI of head E. Visual

Major takeaway Primary angle-closure glaucoma (PACG) is a vision-threatening condition requiring rapid medical care. The characteristic signs and symptoms of PACG are a unilateral red eye with a mid-fixed pupil, corneal edema, eye pain, and blurred vision with nausea. *Tonometry typically shows an increased IOP of ≥30 mm Hg* - appllanation tonometry is a tool that pressure IOP Main explanation This patient has a classic case of primary angle-closure glaucoma (PACG). PACG characteristically affects people >40-years-old, has a female preponderance, and is most common in people of Asian or Inuit descent. Presentation, as in this case, is often acute, with orbital pain, blurred, vision, nausea, vomiting, headache, and the appearance of halos around sources of light. Pupillary dilation due to low light levels (such as in a movie theater) or due to alpha agonists (such as pseudoephedrine and phenylephrine) can trigger an episode of acute angle-closure. Individuals with hyperopia tend to have a shorter eyeball and thus a smaller angle to begin with. Conjunctival vessel dilation, corneal edema, and decreased pupillary constriction to light are common physical examination findings. Intraocular pressure testing will likely reveal a very elevated pressure (≥30 mm Hg), confirming the diagnosis. Not C. This patient has acute angle-closure glaucoma. This condition is caused by obstruction of the trabecular meshwork by the peripheral iris, restricting aqueous out-flow. Dilating the pupil should be avoided as this will worsen the obstruction. Treatment, can in fact, involve medical therapies to constrict the pupil (e.g. pilocarpine). Not D. acute angle-closure glaucoma would show a normal head MRI. This patient requires assessment of their intraocular pressure and rapid treatment. MRI may be useful in diagnosing visual symptoms due to multiple sclerosis. Not E. This patient has acute angle-closure glaucoma, caused by obstruction of the trabecular meshwork by the peripheral iris, restricting aqueous outflow. Chronic glaucoma can cause nerve damage and reduced peripheral vision. This patient is unlikely to have developed these changes yet, and the goal of therapy is to prevent them.

A 51-year-old Asian-American woman comes to the emergency department because of blurred vision, headache, and excruciating right eye pain for an hour. She recently took a large dose of oral pseudoephedrine and used intranasal phenylephrine spray for seasonal allergies before going to the movies. Medical history is noncontributory. She wears corrective lenses for hyperopia and has for many years. Ophthalmic examination shows that her right conjunctival vessels are enlarged and that her right cornea is somewhat opacified. Her left eye appears unremarkable. She has difficulty counting fingers with her left eye covered but not with her right eye covered. Her right eye is poorly reactive to light. Which of the following is the most likely diagnosis? A. Cysticercosis B. Primary angle-closure glaucoma C. Primary open-angle glaucoma D. Retinoblastoma E. Temporal arteritis

Major takeaway Primary angle-closure glaucoma (PACG) typically affects people over the age of 40, has a female preponderance, and is most common in people of Asian or Inuit descent. It is characterized by an acute, painful decrease in visual acuity, vomiting, headache, and the appearance of halos around sources of light. Main explanation This patient has a classic case of primary angle-closure glaucoma (PACG). PACG characteristically affects people over the age of 40, has a female preponderance, and is most common in people of Asian or Inuit descent. Presentation, as in this case, is often acute, with orbital pain, blurred vision, nausea, vomiting, headache, and the appearance of halos around sources of light. Pupillary dilation due to low light levels (such as in a movie theater) or due to alpha agonists (such as pseudoephedrine and phenylephrine) can trigger an episode of acute angle-closure. Individuals with hyperopia tend to have a shorter eyeball and thus a smaller angle to begin with. Conjunctival vessel dilation, corneal edema, and decreased pupillary constriction to light are common physical exam findings. Intraocular pressure testing will likely reveal a very elevated pressure (30 mmHg or higher, normal range 8-21 mmHg). Not A. Cysticercosis is a tissue infection caused by the young form of the pork tapeworm. In some cases, cysticerci may be found in the globe, extraocular muscles, and under the conjunctiva (subconjunctiva). Depending on the location, they may cause visual difficulties that fluctuate with eye position, retinal edema, hemorrhage, a decreased vision or even a visual loss

A 29-year-old man comes to the emergency department because of chest pain and shortness of breath for two days. The difficulty in breathing started suddenly while he was watching television. His temperature is 36.8°C (98°F), pulse is 87/min, respirations are 18/min, and blood pressure is 117/78 mm Hg. Cardiac, pulmonary, and abdominal examinations are noncontributory. ECG shows sinus rhythm. Chest X-ray is obtained. A repeat chest X-ray 6 hours later shows that the pathology has decreased slightly in size. Which of the following is the most appropriate next step in management? A. Admit for pleurodesis B. Discharge the patient with follow-up in 24 hours C. Insert a chest tube D. Observe for another 6 hours E. Perform needle decompression in the second intercostal space, midclavicularline

Major takeaway Primary spontaneous pneumothorax can occur without a precipitating event in patients without known lung disease. If the pneumothorax is small and patients are clinically stable it can be managed conservatively. Main explanation This patient has a primary spontaneous pneumothorax, which occurs in individuals without clinically apparent lung disease. In contrast, secondary spontaneous pneumothoraces occur in individuals with underlying lung disease (e.g. chronic obstructive pulmonary disease (COPD), cystic fibrosis, and lung malignancy). For otherwise healthy, young patients with a small primary spontaneous pneumothorax (<3 cm between the lung and the chest wall on chest X-ray), observation alone may be appropriate. The intrinsic resorption rate is approximately 1-2% a day and is accelerated with the administration of 100% oxygen. Many physicians observe these patients for 6 hours and then repeat the chest X-ray. If the repeat chest X-ray shows no increase in the size of the pneumothorax, the patient can be discharged with follow-up in 24 hours. Air travel and underwater diving (which cause changes in atmospheric pressure) must be avoided until the pneumothorax completely resolves. Not C. Tube thoracostomy is used when the patient is clinically unstable after trauma or in secondary spontaneous pneumothorax where the patient has an underlying and predisposing lung condition. Not E. Needle decompression is an emergency temporizing maneuver for patients with suspected tension pneumothorax. This patient has a pneumothorax, but his vital signs are stable and there are no signs of mediastinal shift on chest X-ray.

A 17-year-old boy from Oklahoma is brought to the emergency department with two days of fever to 39.3°C (102.7°F), myalgia, abdominal pain, and vomiting. A maculopapular rash is noted on the trunk, back, extremities, palms, and soles. He had previously been in good health, with no unusual dietary or travel exposures and no sick contacts. He has been sexually active with multiple partners; he drinks socially but denies use of recreational drugs. He is not taking any medications and has no known drug allergies. Which of the following is the most appropriate next step in management? A. Order a complete blood cell count and a liver function panel B. Order a rapid plasma reagin (RPR) test C. Order a skin biopsy of the rash to obtain samples for immunohistochemical staining D. Order empiric treatment with doxycycline E. Order empiric treatment with penicillin

Major takeaway Typical symptoms of Rocky Mountain spotted fever (RMSF) include fever, headache, myalgia, abdominal pain, vomiting, and rash (often several days later). Since the disease can be fatal if not treated in the first few days, doxycycline should be initiated in any patient with suspected RMSF prior to obtaining confirmatory tests. Main explanation The history of fever and maculopapular rash on the palms and soles is concerning for Rocky Mountain spotted fever (RMSF) infection, especially considering that the patient is from Oklahoma. Since about half of patients with RMSF do not recall a tick bite, the absence of confirmed tick exposure should not be taken to rule out the diagnosis. Given the rapid and potentially fatal course of RMSF, empiric treatment with doxycycline should be initiated immediately in any patient with suspected RMSF. The diagnosis can be confirmed later by polymerase chain reaction, immunohistochemistry, or culture. While secondary syphilis can also present with a rash on the palms and soles, it does not fit the rest of his symptoms and does not require urgent testing or treatment.Not A. While clues such as thrombocytopenia and elevated transaminase levels may be present, laboratory diagnosis is not the most important step in immediate management. Not B. Nontreponemal tests like the RPR test have traditionally been used for initial syphilis screening. In this case, while syphilis is an appropriate consideration in the differential of a rash on the palms and soles, it is not the most important diagnosis to manage immediately. Not C. immunohistochemical staining has good sensitivity (70%) for detecting Rickettsia rickettsii when applied to tissue specimens collected before initiating antibiotic treatment, but confirmatory laboratory tests are not the most important step at this stage. Not E. Based on the patient's symptoms, syphilis is not the most likely diagnosis. In any case, it is not the most important consideration for immediate management.

A 5-week-old infant comes to the emergency department because of continuous, non-bilious projectile vomiting. His mother says he has not been able to keep food down for the past few days and that he seems weaker than before. His temperature is 37.0°C (98.6°F), pulse is 130/min, respirations are 35/min, and blood pressure is 70/45 mm Hg. His abdomen is nontender and nondistended, but a small mass can be palpated below the costal margin. Which of the following is the most likely diagnosis? A. Annular pancreas B. Duodenal atresia C. Hypertrophic pyloric stenosis D. Intestinal atresia E. Malrotation

Major takeaway Pyloric stenosis is likely characterized by continuous non-bilious projectile vomiting, particularly with an olive-shaped mass found in the right upper quadrant/epigastric area. Main explanation The timeline and physical examination findings ("olive mass" in the epigastric area) in this patient are classic for congenital pyloric stenosis. Look for a nursing baby who has not had feeding problems who suddenly develops projective vomiting after feeds. A chemistry panel will reveal a hypochloremic, hypokalemic metabolic alkalosis. Ultrasound will show a donut sign. For pyloric stenosis questions: The best initial test - Abdominal ultrasound Most accurate test - Upper GI series. The best initial treatment - IV fluid and electrolyte replacement. Definitive treatment - Surgical myomectomy of the hypertrophied pylorus. If you are presented with a similar case in a newborn or a patient less than one week old be sure to also consider duodenal atresia and look for bilious vomiting. Not A. This condition presents usually after the first feed and causes the baby to vomit bilious material. Look for a double bubble on X-ray without distal air. Not B. his condition presents usually after the first feed and causes the baby to vomit bilious material. It presents as polyhydramnios in utero. Look for a double-bubble sign on X-ray with no distal air. It's commonly associated with Down syndrome. Not D. This condition presents usually after the first feed and causes the baby to vomit bilious material. Look for double-bubble sign on X-ray with multiple air-fluid levels. Not E. This condition presents usually after the first feed and causes the baby to vomit bilious material. Look for a double-bubble sign on X-ray with normal gas patterns beyond a double-bubble sign.

A 25-year-old woman comes to the emergency department because of unilateral drooping of the right side of her face for the past half hour. She admits to experiencing ear pain for the past few days as well. Physical examination shows small, red vesicles in the right external meatus. Which of the following additional symptoms is most likely to be found in this patient? Elimination tool A. Anosmia B. Hyperacusis C. Ophthalmoplegia of the rightt eye on abduction D. Presbycusis E. Voice hoarseness

Major takeaway Ramsay Hunt syndrome is a complication of shingles that is associated with Bell's palsy. Ramsay Hunt syndrome is characterized by ear pain, ipsilateral facial paralysis, and erythematous vesicles in the ear. Main explanation Bell's palsy is classically described as paralysis of facial muscles due to trauma or inflammation of the facial nerve, cranial nerve VII. There are a few diseases associated with of Bell's palsy, including various herpes viral infections, Lyme disease, human immunodeficiency virus (HIV) infection, and sarcoidosis. Shingles, or herpes zoster, results from a reactivation of latent varicella zoster virus (VZV). It is characterized by a rash consisting of erythematous papules, which progress into grouped vesicles or bullae. One complication of VZV reactivation is Ramsay Hunt syndrome, which is characterized by ipsilateral facial paralysis, ear pain, and vesicles in the auditory canal or external ear. Bell's palsy has been associated with Ramsay Hunt syndrome.The facial nerve is a mixed nerve that contain fibers to salivary glands, afferent fibers for taste, and efferent fibers to the stapedius muscle in the middle ear. The stapedius muscle functions to control the amplitude of sound waves transmitted to the inner ear by stabilizing the stapes. Paralysis of the stapedius will result in hyperacusis, in which normal sounds are perceived to be louder.

A 22-year-old male comes to the emergency department because he is confused and lethargic. His roommates say he was complaining of a severe headache and neck stiffness last night. Vital signs are remarkable for a temperature of 104.3 F, blood pressure of 80/60 mmHg, and heart rate of 120 bpm. Physical examination shows a purpuric rash on the patient's chest. Lumbar puncture reveals a high neutrophil count. Which of the following is the most appropriate treatment to be given to this roommates? A. Acycloir B. Ceftriaxone C. Doxycycline D. Rifampin E. Vancomycin

Major takeaway Rifampin is also indicated for prophylaxis of close contacts of patients with meningococcal meningitis. Symptoms of meningitis include fatigue, fever, headache, and rapidly progressive neck stiffness. Main explanation Rifampin is an antibiotic used to treat a variety of bacterial infections, including tuberculosis, leprosy, and Legionnaire disease. Rifampin is also indicated for prophylaxis of close contacts of patients with meningococcal meningitis. The patient described has acute bacterial meningitis, the cause of which is most likely Neisseria meningitidis, given the patients age, clinical history, and the characteristic *purpuric rash of disseminated intravascular coagulation*. Neisseria meningitidis is a gram negative bacteria that causes meningitis and meningococcemia. It is spread through saliva and respiratory secretions during coughing, sneezing, and kissing. Symptoms of meningitis include fatigue, fever, headache, and rapidly progressive neck stiffness. Death occurs in about 10% of cases.

A 34-year-old woman comes to the emergency department because of severe migraine headache which is unresponsive to tramadol and sumatriptan. Her history indicates that she takes fluoxetine for depression. Soon after being given an injection of meperidine, she develops agitation, diaphoresis, tremor, diarrhea, fever, and incoordination. Which of the following is her most likely diagnosis? A. Thyrotoxic storm B. Neuroleptic malignant syndrome C. Cholinergic crisis D. Epileptic seizure E. Serotonin syndrome

Major takeaway Serotonin syndrome is thought to be due to over-stimulation of the serotonergic system. The first step in management is discontinuing the provocative drug. Main explanation Serotonin syndrome is a preventable, drug-related complication that results from increased brain-stem serotonin activity, usually precipitated by the use of one or more serotonergic drugs. Its clinical presentation consists of autonomic dysfunction, alteration in mental status, and neuromuscular disorder. Early recognition and treatment is important, because this condition is potentially fatal. Management includes withdrawal of causative agents and supportive measures such as hemodynamic stabilization, sedation, temperature control, hydration, and monitoring for complications. Serotonin antagonists, specifically cyproheptadine, have been used, but the documented benefits are purely anecdotal. The best course of action when treating a patient with serotonin syndrome is to remove the offending agents. Then provide emergency supportive care as necessary. There are other adjunct treatments as mentioned in the answer choices. If the syndrome leads to rhabdomyolysis, muscle paralysis and cooling are indicated. A common mnemonic for recognizing the symptoms of serotonin syndrome is HARMED. Hyperthermia Autonomic instability Rigidity Myoclonus Encephalopathy Diaphoresis

A 28-year-old woman comes to clinic because of a 2 month history of amenorrhea. Onset of menses occurred when she was 15 years old, with regular menses until 2 months ago. Physical examination shows a well-appearing patient with a BMI of 20 kg/m2. She denies any recent weight loss, stress, hot flashes, new onset acne, recent gynaecological procedures, or abnormal hair growth. She is sexually active with one partner and uses oral contraceptive pills. Which of the following is the most appropriate initial test? A. Beta-hCG serum measurement B. FSH serum measurement C. Progesterone challenge D. Prolactin serum measurement E. Ultrasound

Major takeaway Serum beta-hCG measurement is the first test to perform in a woman with secondary amenorrhea. Once pregnancy is ruled out, the differential expands and more tests are warranted. Main explanation Secondary amenorrhea is the absence of menstruation in a woman who previously menstruated and now does not. Primary amenorrhea is the absence of menstruation in a woman who has never menstruated. The differential diagnosis for secondary amenorrhea includes pregnancy, obstruction, premature ovarian failure, hyperprolactinemia, hypothalamic amenorrhea, and polycystic ovary syndrome. The first test should be to rule out pregnancy with serum tests for Beta-hCG. Following that, a serum FSH level, Prolactin level, TSH level, and progesterone challenge are reasonable tests for the other conditions. A high FSH level, along with the appropriate history of hot flashes, would be consistent with premature ovarian failure. A history of gynaecological procedures could be indicative of obstruction. A high prolactin level would be consistent with hyperprolactinemia. A high TSH would fit with this as well, as hypothyroidism causing high TRH can cause high prolactin since TRH stimulates prolactin. It is notable that both hypothyroidism and hyperthyroidism can cause amenorrhea. A history of stress, weight loss, or mass symptoms along with a lack of bleed on progesterone challenge could be indicative of hypothalamic amenorrhea. A history of menstrual chaos, hirsutism, alopecia, and/or acne, along with a bleed on progesterone challenge could be indicative of polycystic ovary syndrome.

A 4-year-old girl is brought to the office because of diarrhea for 12 hours. Her mother states that she came home from daycare and developed fever, abdominal pain, and diarrhea containing flecks of bright-red blood and pus. The girl did not ingest any food at daycare, and her diet has been unchanged in the last week. A stool specimen is collected and sent to the lab. Which of the following is the most likely causative organism based on clinical findings? A. Campybacter jejuni B. Enterotoxigenic e coli C. Shigella dysenteriae D. Vibrio cholera E. Yersinia enterocolitica

Major takeaway Shigella dysenteriae is a non-motile gram-negative rod. Humans are the only hosts for Shigella, and it usually causes bloody diarrhea. Transmission is by fecal-oral and hand-to-hand routes. Main explanation S. dysenteriae, spread by contaminated water and food, causes severe dysentery because of its potent and deadly Shiga toxin. Contamination is often caused by bacteria on unwashed hands during food preparation, or soiled hands reaching the mouth. Shigella infection is characterized by high fever (>38.5°C [101.3°F]), abdominal cramps, diarrhea, tenesmus, and polymorphonuclear leukocytes on a methylene blue stain of the stool. The characteristic small-volume, stools mixed with *blood and mucus are present in approximately one-half of the children with shigellosis*. The WHO recommends that all suspected cases of shigellosis based on clinical features be treated with effective antibiotics. Commonly used antibiotics include beta-lactams, cephalosporins, and macrolides. However, infection with Shigella generally is self-limited; with the average duration of GI symptoms in untreated pediatric Shigella gastroenteritis being approximately seven days. Despite this, in the absence of specific antibiotic treatment, children with Shigella gastroenteritis shed the organism for up to four weeks, facilitating its spread to others. Not A. Campylobacter jejuni also causes bloody diarrhea, but is usually due to ingestion of contaminated foods or water, with animals as the main reservoir. . There is no history of contaminated food ingestion. Not E. Yersinia enterocolitica typically occurs in young children, especially at day-care centers. Symptoms include fever, abdominal pain, and (possible bloody) diarrhea. Onset is 4-7 days after exposure and may last 1-3 weeks or longer. Transmission is through ingestion of contaminated food and water. Yersiniosis has a much lower incidence than shigellosis

A 35-year-old male comes to the emergency department because of a stiff neck, severe headache, and vomiting. He states that these symptoms developed suddenly, while he was out for his daily run. Medical history is noncontributory. His temperature is 37.3°C (99°F), pulse is 87/min, respirations are 18/min, and blood pressure is 117/78 mm Hg. Examination shows a fit appearing man. A non-contrast CT-scan is obtained. Which of the following is the most likely diagnosis? A. Contusion B. Epidural hematoma C. Subarachnoid hematoma D. subdural hematoma E. meninigitis

Major takeaway Subarachnoid hemorrhage (SAH) is bleeding into the subarachnoid space, the area between the arachnoid membrane and the pia mater surrounding the brain. While SAH commonly occurs in older people, it can affect people of any age. Main explanation A subarachnoid hemorrhage (SAH) is bleeding into the subarachnoid space—the area between the arachnoid membrane and the pia mater surrounding the brain. This may occur spontaneously, usually from a ruptured cerebral aneurysm, or may result from head injury. Signs and symptoms of SAH include a severe headache with a rapid onset ("thunderclap headache"), vomiting, confusion or a lowered level of consciousness, and sometimes seizures. In general, the diagnosis is confirmed with a CT scan of the head. If there is a high index of suspicion and the CT scan is normal, then lumbar puncture should be performed. On lumbar puncture the fluid looks yellow due to xanthochromia (presence of oxidized RBCs). CT shows blood pooling in the cisterns.While SAH commonly occurs in older people, it can affect people of any age. It can also be mistaken for a number of other diagnoses including migraine, cluster headaches, and meningitis so a high degree of clinical suspicion is necessary.

A 53-year-old man comes to his primary care provider's office because of a painless genital ulcer and a maculopapular rash affecting his palms and soles for the past 3 months. A diagnosis of syphilis is made. His current medications include aspirin and warfarin. The patient reports that he has a penicilllin allergy. Which of the following is the next best step in management? A. Penicillin desensitization B. Azithromycin C. Glanciclovir D. Rifampin E. Doxycycline

Major takeaway Syphilis is a sexually transmitted disease caused by Treponema pallidum, a gram negative spirochete. Patients with penicillin allergies should receive doxycycline. Main explanation Syphilis is a sexually transmitted disease caused by Treponema pallidum, a gram negative spirochete. The first stage of syphilis is characterized by a painless genital ulcer. The second stage of syphilis is characterized by a maculopapular rash affecting the palms and soles. The third stage of syphilis include systemic symptoms such as Argyll Robertson pupils, aortitis, and tabes dorsalis. The mainstay of treatment for syphilis is penicillin G. However, patients with serious documented allergies to penicillin should be treated alternatively with doxycycline or tetracycline as per the center of disease control and prevention guidelines. These medications should be avoided in pregnancy as they can damage the teeth of an unborn fetus. Hence, pregnant patients should be desensitized to penicillin first prior to penicillin therapy.

A 13-year-old boy is brought to the emergency department because of acute onset of severe pain in his left scrotum for the past 3 hours. He also reports nausea and vomiting. Physical examination shows a high-riding left testicle with a horizontal lie, absent cremasteric reflex, and an enlarged, erythematous, and swollen scrotum. Which of the following is the definitive step in management of this condition? A. Immediate surgery for detorsion and bilateral orchipexy B. Rotation of testis along its axis to relieve torsion and doppler ultrasound to access for return of bloodflow C. Start antibiotics for this episode of epididymitis D. Administration of analgesia and observation E. Immediate surgery for detorsion and unilateral orchiopexy

Major takeaway Testicular torsion is a medical emergency that requires prompt surgical treatment within 6 hours to prevent loss of the testicle. Symptoms include a high-riding, horizontal testicle in a swollen scrotum with loss of cremasteric reflex. Main explanation Testicular torsion occurs as a result of the testis rotating about the spermatic cord. This strangulates the blood supply and leads to ischemia and necrosis of the testis if proper surgical management is not done. Testicular torsion is commonly a result of a congenital malformation where the testicle is not adequately adhered to the scrotal wall. Testicular torsion occurs in 1 per 25,000 males and most commonly occurs during the adolescent puberty phase. However, neonatal testicular torsion is also fairly common. Testicular torsion is characterized by acute onset severe scrotal pain. Physical examination may show a high-riding testicle with a horizontal lie within a swollen and erythematous scrotum. The cremasteric reflex will be absent. While doppler ultrasound can be utilized to access testicular blood flow, it should only be utilized in cases where diagnosis is unclear. Testicular torsion is a surgical emergency and surgery should be done within 6 hours of onset. Bilateral surgical orchiopexy (fixation of the testes within the scrotum) is needed to prevent future episodes of torsion and to preserve fertility. Not D. Bilateral orchiopexy> unilateral bc helps to prevent torsion of contralateral testis in future!

A 2-week-old girl is brought to the emergency department by her mother because of a blue coloring to her skin. The mother says that her baby seems "purple," especially her fingers and toes, and looks extremely blue when crying. On physical examination the sleeping baby has mild cyanosis of the face and trunk, but moderate cyanosis of the extremities. Which of the following is the most common cause of this patient's condition within the first few weeks of life? A. Atrial septal defect B. Patent ductus arteriosus C. Tetralogy of Fallot D. Transposition of the great vessels E. Ventricular septal defect

Major takeaway Tetralogy of Fallot is a congenital heart defect. It is the most common cause of cyanosis within the first few weeks of life. The skin becomes bluish because of the malformed right-to-left shunt. Infants also have worsening cyanosis with agitation, difficulty feeding, and failure to gain weight. Main explanation Tetralogy of Fallot is a congenital heart defect that is present at birth. It is the most common cause of cyanosis within the first few weeks of life. The skin becomes bluish because of the malformed right-to-left shunt. Infants also have worsening cyanosis with agitation, difficulty feeding, and failure to gain weight. When affected babies have a bowel movement or cry, they may develop a "tet spell" where they turn blue, become limp, have difficulty breathing, and can lose consciousness. Patients may also have clubbing of the fingers and toes or even polycythemia. Other symptoms may include a heart murmur and easy tiring upon breastfeeding. The four components of the teratology are (1) ventricular septal defect, (2) overriding aorta, (3) infundibular pulmonary stenosis, and (4) right ventricular hypertrophy. Risk factors include a mother who uses alcohol, had diabetes, is over the age of 40, or gets rubella during pregnancy. It is also often associated with Down syndrome. Treatment typically involves open heart surgery during the first year of life.

A 35-year-old woman with a past history of asthma comes to the emergency department in respiratory arrest and intubation is requested. There is mist in the endotracheal tube when passed through the vocal chord. End tidal CO2 is 45 mm Hg. Pulse oximetry in room air shows an oxygen saturation of 90%. No breath sounds are heard over the epigastric area, but weak sounds are heard over the left lung and breaths can be clearly heard over the right lung. Her teeth are 23 cm. Which of the following is the most appropriate next-step in management? A. Consult with anesthesia B. Extubate patient and re-intubate C. Pull back tube 1-2 cm and re-check to confirm

Major takeaway The best means of assessing correct placement of an endotracheal tube is by listening to breath sounds and then confirming with a plain film X-ray. Asymmetrical breath sounds on examination suggests intubation of the right main-stem bronchi. An appropriate response is to pull back the tube 1-2 cm and re-assessing. Main explanation There is a lot of debate about what the end tidal CO2 and pulse oximetry saturation concentrations should be for an endobronchial intubation (intubation of the right main-stem bronchi) and a supraglottic intubation (too shallow). The best investigation to evaluate success and proper placement of an endotracheal intubation is through listening to breath sounds and confirming with an X-ray. Be aware that right main-stem bronchi intubations happen more frequently in women than men. The depth of the endotracheal tube (ETT) for women is on average 20-21 cm, compared to the 22-23 cm average for men.Because of the weak breath sounds on the left lung and strong breath sounds on the right lung, the most appropriate next-step in management is pulling back the tube, 1-2 cm, and re-checking for confirmation.

A 70-year-old woman comes to the hospital because of pneumonia. The patient had normal mental status upon admission, and throughout the course of the day. In the evening, the patient appears to be agitated with an altered mental status. The patient is combative with nursing staff, and makes violent gestures while trying to be examined. She is making comments about a small animal being in her room. There is no history of drug or alcohol abuse. A blood glucose is obtained, and is 98 mg/dL. Which of the following is the next best step in management in this patient? A. Dihphenhydramine B. Haloperidol C. Chlordiazepoxide D. Diazepam E. Physical restraints

Major takeaway The first step in management of a patient with delirium and psychomotor agitation is a typical antipsychotic. Physical restraints may be a useful adjunct. Main explanation This patient is experiencing delirium with psychomotor agitation. She requires chemical restraint. Delirium is altered state of consciousness due to drugs, infection, hypoxia, or central nervous system abnormalities. It is often quickly reversible once the underlying cause is identified. Key features of delirium include altered level of consciousness with inattentiveness and confusion, change in cognition that is not caused by preexisting dementia, changes in cognition develop quickly and fluctuate over course of day, and changes are related to disease, medication, or drug use. Other features include psychomotor agitation or retardation, disturbance of sleep patterns, emotional instability, and visual hallucinations. *Antipsychotic medications are the treatment of choice to decrease agitation acutely*. In particular haloperidol is the most-studied antipsychotic in delirium treatment because of its high potency, low sedative effect, few anticholinergic side effects, minimal cardiovascular side effects, no active metabolites, and multiple administration routes. E. Physical restraints are recommended as an adjunctive agent when treating delirium, but not the first step in management. It is not recommended to physical restrain a patient without chemical restraints because the patient can sustain worsening symptoms. A *patient in restraints is at risk for rhabdomyolysis, strains, sprains, and fractures.*

An 26-year-old man comes to the emergency department because of scrotal pain. He has had progressive swelling and tenderness to the right testicle for several days. It has been associated with a low-grade fever, and mild burning with urination. He admits to having unprotected sexual intercourse with multiple partners. Physical examination is remarkable for a tender, swollen right testicle. The cremasteric reflex is intact bilaterally. There is no urethral discharge. A scrotal ultrasound is performed that shows increased blood flow to the right testicle. Which of the following is the most appropriate management in this patient? A. Outpatient referral to urology B. Ceftriaxone and doxycycline C. Ciprofloxacin D. Manual detorsion E. Emergent orchiopexy

Major takeaway The most common cause of epididymitis in sexually active patients less than 35 is Chlamydia trachomatis or Neisseria gonorrhoeae. Treatment is within intramuscular ceftriaxone (covers Neisseria gonorrhoeae) and 10 days of oral doxycycline (covers Chlamydia trachomatis). Main explanation Epididymitis is a condition that results in discomfort or pain of the epididymis. The most common cause of epididymitis in sexually active patients less than 35 is Chlamydia trachomatis or Neisseria gonorrhoeae. Treatment is within intramuscular ceftriaxone (covers Neisseria gonorrhoeae) and 10 days of oral doxycycline (covers Chlamydia trachomatis). Treatment for sexually transmitted diseases will typically involve empiric coverage for both Chlamydia trachomatis and Neisseria gonorrhoeae because culture results are infrequently available at the time of diagnosis. If cultures are available, narrowed antibiotic choices may be used. Note that his medication combination also covers Escherichia coli, the other most common cause of epididymitis and is the same treatment for pelvic inflammatory disease in females.

A 15-year-old girl comes to the emergency department because of left knee pain. She states the pain has been increasing for the past 2 weeks. She also reports fevers, diffuse joint pain, and a history of sickle cell disease. Her temperature is 38.3°C (100.9°F), pulse is 88/min, respirations are 20/min, and blood pressure is 122/82 mm Hg. Physical examination shows the left knee is erythematous, warm, swollen, and tender to palpation over the tibial prominence. Radiographs of the knee show soft tissue edema and a large radiolucency in the proximal tibia. Which of the following is the most likely infectious organism? A. Group B streptococcus B. Haemophilus influenzae C. Legionella pneumophila D. Pseudomona aeruginosa E. Staphylococcus aureus

Major takeaway The most common cause of osteomyelitis in pediatric patient groups including patients with sickle cell disease is Staphylococcus aureus. The characteristic radiographic changes of osteomyelitis include soft tissue changes, loss of normal fat pad planes, and effusions in adjacent joints. Main explanation Osteomyelitis refers to bony inflammation and is almost always due to infection, typically bacterial. It is particularly common between the ages of 2 and 12 and 3x more common in males than females. Even among pediatric patients with sickle cell disease, Staphylococcus aureus is the MCC of osteomyelitis in all demographic groups. After S. aureus, Escherichia coli is relatively common in intravenous drug users, and Haemophilus influenzae is seen among neonates. On a plain film, the earliest changes are seen in adjacent soft tissues +/- muscle outlines with swelling and loss/blurring of normal fat planes. An effusion may be seen in an adjacent joint. Generally, osteomyelitis must extend at least 1 cm and compromise 30 to 50% of bone mineral content to produce noticeable changes in plain radiographs. Early findings may be subtle, and changes may not be obvious until 1 week in children and 2 weeks in adults. After this time, other changes such as regional osteopenia, periosteal reaction, focal bone lysis, new bone apposition, and sclerosis may occur.

A 54-year-old woman comes to the emergency department because of vertigo for the past 20 minutes. She has a one-year history of attacks of vertigo, left ear fullness, left ear hearing loss, and tinnitus. One month ago, she was treated with oral steroids by another physician and her symptoms improved. She has subsequently had three more vertigo attacks with similar symptoms in her left ear. Physical examination shows Rinne and Weber tests are consistent with sensorineural hearing loss in the left ear. No nystagmus is present. Which of the following is the most likely diagnosis? A. Benign paroxysmal positional vertigo B. Labyrinthitis C. Meniere disease D. Otitis Media E. Vestibular Neuronitis

Major takeaway Think of Meniere disease when a patient presents with recurrent episodes of rotatory vertigo associated with hearing loss, aural fullness, and tinnitus. Main explanation Meniere disease, also known as endolymphatic hydrops, is an idiopathic disorder of the inner ear associated with increased endolymphatic pressure. The clinical features of Meniere disease include recurrent episodes of vertigo, aural fullness, sensorineural hearing loss, and tinnitus. Not all symptoms need to be present for the diagnosis and *vertigo is the most common* presenting complaint. The vertigo may be brief, lasting minutes to hours, or may be longstanding and be associated with nausea and vomiting. At least two episodes of vertigo of at least 20 minutes in duration are required to make a diagnosis. Fluctuating sensorineural hearing loss affects the low frequency tones and tends to worsen as the disorder progresses with permanent hearing loss occurring in most patients.Management is focused on lifestyle change, particularly a low-sodium diet. Not A. Benign paroxysmal positional vertigo is a disorder arising from otoliths in the inner ear. Symptoms are repeated brief periods of vertigo, which occur with turning of the head. Each episode of vertigo typically lasts less than one minute, although associated nausea may last much longer. Not B. Labyrinthitis occurs due to inflammation of the inner ear and is characterized by severe vertigo, nausea and vomiting, tinnitus, hearing loss, and aural fullness. It often occurs in the setting of a viral or bacterial infection and fever may also be present. Unlike Meniere disease labyrinthitis is not usually episodic. Not E. Vestibular Neuronitis Vestibular neuritis is characterized by vertigo, and may be accompanied by associated nausea and vomiting. It is due to inflammation of the inner ear but unlike labyrinthitis auditory function is preserved.

A 68-year-old man comes to the clinic because of swelling and tenderness of his left thigh. His temperature is 37.2°C (99.0°F), pulse is 85/min, respirations are 18/min, and blood pressure is 135/80 mm Hg. There is erythema upon inspection and a nodular cord upon palpation. Which of the following is the most appropriate next step in management? A. CTA B. Amoxicillin 500mg C. Fondaperinux 2.5mg D. Duplex ultrasound E. Hypercoagulability testing

Major takeaway Thrombophlebitis is inflammation and thrombus formation, usually in the lower superficial veins. It can be diagnosed clinically, but is best characterized by doppler ultrasound to rule out concurrent deep vein thrombosis. Main explanation Thrombophlebitis is a combined inflammation and thrombus of the veins. Superficial thrombophlebitis is more common in the lower extremities. Thrombosis is associated with damage to the vessel intimal layer, stasis or turbulent flow, and/or increased coagulability (i.e. Virchow triad). The thrombophlebitis presents as erythema and tenderness with a palpable cord due to the thrombus in the superficial vein. Low grade fever may be present, even in uncomplicated cases. Complicated thrombophlebitis is used to described recurrent events, suppurative thrombophlebitis, thrombophlebitis with accompanying thromboembolism, or migratory thrombophlebitis, which is often associated with underlying malignancy. Diagnosis of thrombophlebitis is clinical, but duplex ultrasound is used to characterize the location and extent of the thrombosis. The ultrasound shows thickening of the venous wall and perivenous edema. Duplex ultrasound is especially important in determining concurrent deep vein thrombosis. Uncomplicated thrombophlebitis can be managed supportively with elevation, compresses, and non steroidal anti-inflammatory drugs. Not A. CTA would be indicated if there were suspicion of pulmonary embolism. However, there is no need for such further imaging if duplex ultrasound confirms superficial thrombophlebitis Not B. This would be a suitable treatment for erysipelas or nonpurulent cellulitis. Although these infections manifests as erythema, warmth, and edema, a palpable nodular cord suggests that thrombophlebitis should first be ruled out. Not C. Low molecular weight heparin is indicated for some patients with thrombophlebitis of the superficial veins, but only when they are at increased risk of thromboembolism. The size and depth of the thrombus should first be evaluated with ultrasound. Not E. Screening for an abnormality in the coagulation profile is not indicated following a single episode of thrombophlebitis. However, it should be considered in recurrent cases.

A 24-year-old woman comes to the emergency department because of urinary frequency, burning sensation on urination, and pelvic tenderness. A urine culture shows a lactose-positive, gram-negative rod. Further testing shows a negative catalase test and positive indole test. The patient is prescribed a paraaminobenzoic acid (PABA) analog that functions as a competitive inhibitor of dihydropteroate synthetase and is discharged home. Four days later, she returns with complaints of a burning rash covering her entire chest, abdomen, and back. Which of the following conditions is the most concerning for this patient? A. Penicillin allergy B. Stevens-Johnson syndrome C. Toxic epidermal necrolysis D. Type I reaction to sulfamethaxzole E. Type IV reaction to sulfasalazine

Major takeaway Toxic epidermal necrolysis (TEN) is an extremely severe and sometimes fatal drug reaction that may occur in patients exposed to sulfa drugs, penicillins, anti-epileptic drugs or allopurinol. Main explanation Stevens-Johnson syndrome (SJS) and toxic epidermal necrolysis (TEN) are severe skin reactions characterized by extensive necrosis and detachment of the epidermis. These two conditions are categorized based on the extent of necrosis; SJS refers to cases with less than 10% epidermal involvement, while TEN refers to cases with over ≥30% involvement. SJS/TEN overlap refers to cases with 10-30% involvement. Medications are the predominant underlying cause of SJS/TEN. TEN is the more dangerous condition with a 30% mortality rate. The *most commonly implicated drugs are allopurinol, aromatic anticonvulsants, sulfonamides, and "oxicam" non-steroidal anti-inflammatory drugs (NSAIDs)*. Signs of SJS/TEN include fever, influenza-like symptoms, erythematous macules with atypical target lesions, and sloughing of the skin as the disease progresses. A burning rash is often a sign of an impending hypersensitivity reaction. A possibly useful acronym for remembering drugs that can cause SJS/TEN is: *Steven Johnson* has *epileptic Allergy* to *Sulfa drugs and Penicillin*. -Steven Johnson (to remember for what condition this is a mnemonic) -Epileptic = carbamezapine, valproate, zonisamide, lamotrigine, phenobarbital -All(ergy) = Allopurinol -Sulfa drugs = sulfamethoxazole-trimethoprim, furosemide, hydrochlorothiazide, acetazolamide, others -Penicillin = Penicillin and penicillin derivatives

A 48-year-old man comes to the emergency department after collapsing at his home. His wife states that he returned from a backpacking trip along the Mississippi River one month ago. He had developed an ulcer on the back of his neck with some fever and lymphadenopathy after a tick bite during his trip but refused to see a physician because the ulcer was painless. Currently, his temperature is 39.6°C (103.3°F), pulse is 110/min, respirations are 23/min, and blood pressure is 68/36. Physical examination shows widespread purpura across the patient's entire body. There is a white scar on the back of his neck where the ulcer had been previously. The following serological values are obtained:Hemoglobin: 13 mg/dLHematocrit: 40%Leukocytes: 30,000/mm3Platelets: 29,000/mm3Alanine aminotransferase: 130 U/LAspartate aminotransferase: 165 U/LBlood urea nitrogen: 39 mg/dLCreatinine: 2.4 mg/dLWhich of the following is the most appropriate diagnosis? A. Anthrax B. Mucocutaneous leishmaniasis C. Streptococcall toxic shock syndrome D. Tertiary syphilis E. Tularemia

Major takeaway Tularemia is initially characterized by a painless ulcer that progresses to lymphadenopathy and fever. If untreated, it can progress to sepsis. Transmission of F. tularensis occurs via exposure to infected animals (rabbits) or vectors (Dermacentor ticks). Main explanation Francisella tularensis is a highly virulent, gram-negative, pleomorphic coccobacillus. It is distributed across the United States and Europe; most cases in the United States occur in the Midwest. It has a plethora of vectors, the most important being ticks of the Dermacentor genus. The most common reservoirs of F. tularensis are wild rabbits, deer, and rodents. The incubation of F. tularensis is usually 3-5 days. The most common manifestation of infection is the development of a soft, painless ulcer at the site of contact with the vector or infected animal. The ulcer later progresses to a scar. Fever and lymphadenopathy accompany ulcer formation. F. tularensis infection can also involve the eye (unilateral conjunctivitis, photophobia), oropharynx (ulcerative/exudative stomatitis and pharyngitis), and lungs (pleuritic chest pain, dyspnea). Ulceroglandular disease can progress to septic tularemia if untreated, with features such as shock, disseminated intravascular coagulation (DIC), and multiple organ failure. Diagnosis usually involves testing serum samples for anti-F. tularensis IgM or IgG. Culture should be performed cautiously due to the organism's high virulence. Treatment of severe tularemia involves an aminoglycoside such as streptomycin (adults) or gentamicin (children). Milder cases in adults can be treated with oral agents such as ciprofloxacin or doxycycline. B. Mucocutaneous leishmaniasis can be found in the Middle East and Africa (Old World, Leishmania topica) or in Central/South America (New World, L. mexicana & L. braziliensis). Transmission occurs via sandfly bites. A slow-healing, necrotic ulcer forms at the bite site and may be complicated by granulomas and bacterial superinfection.

A 53-year-old man is brought to the clinic because of a severe headache and decreased mental status. 20 minutes later patient is obtunded. Medical history is contributory for hypertension for the past 14 years. Ophthalmic examination shows anisocoria and a blown pupil. CT findings contraindicate the use of thrombolytics. Which of the following is the most likely diagnosis? A. Cerebellar tonsillar herniation B. Cingulate herniation C. Dandy-Walker malformation D. Type II Arnold Chiari malformation E. Uncal herniation

Major takeaway Uncal herniation occurs when the uncus of the medial temporal lobe herniate through the tentorium and compress the midbrain. Main explanation *Brain herniation is a side effect of very high pressure within the skull* that occurs when a part of the brain is squeezed across structures within the skull. Herniation can be caused by traumatic brain injury, intracranial hemorrhage, or brain tumor. Herniation can also occur in the absence of high ICP when mass lesions such as hematomas occur at the borders of brain compartments. In such cases, local pressure is increased at the place where the herniation occurs, but this pressure is not transmitted to the rest of the brain, and therefore does not register as an increase in ICP. Therefore, extreme measures are taken in hospital settings to prevent the condition by reducing intracranial pressure, or draining an hematoma. This patient is likely experiencing a subarachnoid hemorrhage leading to increased intracranial pressure. This increase in intracranial pressure can cause parts of the brain to herniate under or through different parts of the meninges. This patient's *aniscoria is indicative of an uncal herniation.* Notably, the use of thrombolytics are contraindicated in hemorrhagic cerebrovascular accidents. Uncal herniation occurs when the uncus of the medial temporal lobe herniate through the tentorium and compress the midbrain. B. Cingulate herniation occurs when the cingulate gyrus herniates beneath the falx cerebri. This can lead to compression of the anterior cerebral artery.

A 75-year-old man comes to the emergency department because of uncontrollable bleeding from his nose for the past 20 minutes. He says that blood started spontaneously dripping from his left nostril while he was reading a book. He says that he tried tilting his head back and placing tissues into his nares but it didn't stop the bleeding. He has no history of trauma preceding the event. Past medical history includes hypertension and atrial fibrillation. Current medications include hydrochlorothiazide, metoprolol, and warfarin. His heart rate is 68/min, respiratory rate is 12/min, and blood pressure is 180/100 mmHg Physical examination shows clotted blood in both nares and visible clots in the posterior oropharynx. There is a slow and active bleed from both nares. Laboratory analysis reveals a hematocrit of 45% and an international normalized ratio (INR) of 2.5. Which of the following is the most appropriate next step in management? A. Place anterior nasal packing B. Reduce BP to 120/80 C. Administer oral vitamin K1 D. Cauterizebleeding vessels with silver nitrate E. Place firm pressure on the cartilaginous portion of the nose for thirty minutes

Major takeaway Warfarin inhibits production of vitamin K-dependent clotting factors and can lead to spontaneous epistaxis. First line treatment for epistaxis includes continuous, firm pressure on the cartilaginous portion of the nose for up to 15 minutes. Main explanation Initial management of epistaxis is non-invasive, manual pressure on the high cartilaginous portion of the nose. Firm pressure should be maintained for up to 15 minutes without "peeking or checking" to see if the bleeding has stopped. It is often helpful to have the patient watch a clock closely. Head position should be maintained neutral or forward, allowing bleeding to collect in the nasal passage and clot, as opposed to traveling down the posterior nasopharynx and into the esophagus or trachea. About 90% of nosebleeds are "anterior" involving Kiesselbach plexus and can be managed without specialty care. This patient has not yet tried direct pressure and this should be attempted before more aggressive options are explored. It is important to consider the potential cause of the epistaxis. Warfarin is an anticoagulant agent that inhibits the production of vitamin K-dependent clotting factors (II, VII, IX, X, and proteins C and S). It is not uncommon to have spontaneous epistaxis on this medication and warrants a check of the INR (international normalized ratio) in the patient. His hematocrit is normal and his INR is therapeutic (between 2-3), thus giving the clinician information about coagulation. He does not need immediate reversal of anticoagulation. Not A. Anterior packing with merocel gauze or commercial anterior packing products is commonly used and effective means of hemorrhage control in anterior epistaxis. Often packing is used in conjunction with a topical vasoconstrictor such as cocaine, epinephrine, or oxymetazoline for vasoconstriction. This is again second line treatment. Not B. Reduction of blood pressure in epistaxis has not been shown to influence hemorrhage control. In fact, hypertension is often blamed for causing epistaxis because many patients present with hypertension. Focus should be on control of bleeding and patient reassurance. Not C. Vitamin K1 should be used for patients taking warfarin who have bleeding and supratherapeutic INR levels (INR >5). This patient has an appropriate INR and there are few recommendations for management in this scenario. Minimal bleeding, such as in this patient, poses little threat for significant blood loss.

A 3-year-old girl is brought to the emergency department because of fever, vomiting, and nausea. Physical examination shows a painless abdominal mass, that extends from the flank to the midline, on palpation. Abdominal ultrasonography is obtained and shows that part of the mass extends to the inferior vena cava. Further biopsy indicates blastemal, stromal, and epithelial cells. Which of the following is the most likely diagnosis? A. Mesoblastic nephroma B. Noonan syndrome C. Renal cell carcinoma D. Renal cyst E. Wilms tumor

Major takeaway Wilms tumor is a type of cancer of the kidneys and is characterized by palpable abdominal mass, fever, nausea, vomiting, hematuria, and hypertension. Histologic examination shows a tumor arising from the fetal kidney and is characterized by *blastemal, stromal, and epithelial cells* (triphasic tumor). Main explanation Wilms tumor, or nephroblastoma, is a type of cancer of the kidneys that occurs in children and rarely in adults. Approximately 500 cases are diagnosed in the U.S. annually. The majority (75%) occur in otherwise normal children; a minority (25%) are associated with other developmental abnormalities. It is highly responsive to treatment, with about 90% of patients surviving at least five years. Wilms tumor is characterized by a* painless palpable abdominal mass, fever, nausea, vomiting, hematuria (in 20% of cases), and hypertension* (especially if synchronous or metachronous bilateral kidney involvement). Histologic examination shows a tumor arising from the fetal kidney and is characterized by blastemal, stromal, and epithelial cells (triphasic tumor). Wilms tumor is *associated with multiple disease *processes like neurofibromatosis, WAGR syndrome (Wilms tumor, aniridia, genitourinary abnormalities, and cognitive impairment), Beckwith-Wiedemann syndrome, and Denys-Drash syndrome.

A 42-year old homosexual man comes to the emergency department complaining of headache and body aches for two days. Ten days ago he was vaccinated against yellow fever, since he is planning to travel to Africa two weeks from now. His last visit was 10 days ago, and he denies any past medical history. His temperature is 38.1 ºC, pulse is 89/min, respirations are 20/min, and blood pressure is 130/80 mm Hg. Initial laboratory results are shown below.What is the most likely cause of this patient's symptoms? A. Acute yellow fever B. Autoimmune hepatitis C. Enterovirus infection D. Vaccine reaction E. Viral hepatitis

Major takeaway Yellow fever causes hepatic injury, with proteinuria, *ESR close to zero and leukopenia*. The last two parameters may be useful to differentiate vaccine reaction from acute infection, after vaccination. Main explanation Acute adverse reactions are common after vaccination. The yellow fever vaccine is prepared with live-attenuated virus, and infection may occur in immunosuppressed patients. However, acute yellow fever is characterized by leukopenia, not leukocytosis, as well as very low erythrocyte sedimentation rate, typically close to zero, and proteinuria. This patient seems to be experiencing an adverse reaction from the vaccine, and no other investigation is required. Not A. Yellow fever vaccine is prepared with live attenuated virus, and may cause infection in immunosuppressed patients. Although HIV screening should be indicated for this patient, a characteristic finding of yellow fever is an ESR close to zero. Also, these patients present proteinuria early in disease, due to acute tubular necrosis. Not E. Homosexual males should be screened for viral hepatitis as well, but viral hepatitis typically causes extense hepatic injury, with aspartate aminotransferase/alanine aminotransferase more than 1.000 UI/L. Further, nothing in the history makes one suspect acute viral hepatitis.

A 2-year-old child comes to the emergency department because of fever, harsh cough, and difficulty breathing for a day. His mother states that her 5-year-old brother who attends a local school was recently ill with similar symptoms. Medical history includes up-to-date immunizations. Her temperature is 37.4ºC (98.0ºF), pulse is 160/min, respirations are 64/min, and blood pressure is 90/62 mm Hg. Pulmonary auscultation shows loud stridor. A frontal x-ray of the upper chest airways is obtained. Which of the following is the most likely diagnosis? A. Epiglottitis B. Asthma C. Croup D. Airway foreign body E. Bronchiolitis

Major takeaway Croup is a common infection in infants that produces a characteristic barking cough and stridor. On neck or chest X-ray the "steeple sign" showing tracheal inflammation is characteristic of croup. Main explanation Croup or tracheolaryngitis is caused by an infection of the upper respiratory tract, typically viral in nature. This infection leads to inflammation and swelling of the trachea and upper airways, which in turn leads to a "barking" cough, stridor, and potentially fever. In tracheolaryngitis, neck X-ray characteristically shows a "steeple sign" in the trachea which provides clear evidence of tracheal swelling. This investigation also helps to eliminate more serious diagnoses, such as the aspiration of a foreign body into the trachea. Treatment of croup involves giving oral dexamethasone in order to reduce the swelling in the trachea. Following this, treatment is supportive and the condition typically resolves after a few days. No A. While epiglottitis has similar symptoms, and would present with the swollen upper airways , a cough is not common with epiglottitis. The steeple sign on chest X-ray is characteristic of croup.

A 5-year-old boy is brought to the emergency department because of recurrent seizures. He has moderate cognitive impairment and severe motor delay. MRI is obtained and shows abnormal brain development (excessively folded and fused gyri and an abnormally thick cerebral cortex) in a generalized distribution and is most severe in the perisylvian regions, as well as reduced volume of white matter, and ventriculomegaly. Which of the following is the most likely diagnosis? A. Bilateral frontal polymicrogyria B. Bilateral frontoparietal polymicrogyria C. Bilateral generalized polymicrogyria D. Bilateral parasaggital parieto-occipital polymicrogyria E. Bilateral perisylvian polymicrogyria

Major takeaway Bilateral generalized polymicrogyria (BGP) is most severe in the perisylvian regions, but occurs in a generalized distribution. Associated factors include a *reduced volume of white matter and ventriculomegaly*. BGP shows excessively folded and fused gyri of an abnormally thin cerebral cortex, and an absence of the normal six-layered structure. Main explanation Polymicrogyria (PMG) is a condition that affects the development of the human brain by multiple small gyri creating excessive folding of the brain leading to an abnormally thick cortex. This abnormality affects either one region of the brain or multiple regions.The time of onset has yet to be identified, however it has been found to occur before birth in either the earlier or later stages of brain development. Early stages include impaired proliferation and migration of neuroblast, while later stages show disordered post-migration development. The symptoms experienced differ depending on what part of the brain is affected. There is no specific treatment to get rid of this condition, but there are medications that can control the side effects such a seizures, delayed development or weakened muscles as some of the noted effects. This patient has bilateral generalized polymicrogyria (BGP). BGP is most severe in the perisylvian regions, but occurs in a generalized distribution. Associated factors include a reduced volume of white matter and ventriculomegaly. BGP shows excessively folded and fused gyri of an abnormally thin cerebral cortex, and an absence of the normal six-layered structure. Not A. Bilateral frontal polymicrogyria (BFP) is a type of polymicrogyria, which is a developmental malformation of the brain with excessive number of small gyri on the brain surface. BFP is characterized by cognitive impairment, motor delay, spastic quadriparesis, and epilepsy. Not B. Bilateral frontoparietal polymicrogyria (BFPP) has a gene identified linked to the cause, which is GPR56. BFPP patients have atrophy of the cerebellum and brain stem and bilateral white matter abnormalities. BFPP is characterized by estopia (feature of severe static encephalopathy), global development delay, pyramidal signs, cerebral signs, and seizures. D. Bilateral parasaggital parieto-occipital polymicrogyria (BPOP) is located in the parasagittal and mesial regions of the parieto-occipital cortex. This form is associated with IQ scores that range from average intelligence to mild cognitive impairment, and seizures. E. Bilateral perisylvian polymicrogyria (BPP) is characterized by its location; the cerebral cortex deep in the sylvian fissures is thickened and abnormally infolded, as well as the sylvian fissures extending more posteriorly up to the parietal lobes and more vertically oriented.

A 3-year-old boy with Down syndrome comes to the clinic because his mother has noticed enlarged lymph nodes and easy bruising when he plays outside. He seems fatigued and irritable recently. Physical examination shows several bruises on his extremities and petechiae on his trunk. Complete blood count shows lymphopenia, anemia, and thrombocytopenia Peripheral smear reveals the following: Which of the following, in addition to Down syndrome , is also associated with increased risk of this condition? A. Beckwith-Wiedemann syndrome B. Neurofibromatosis type 1 C. Tuberous sclerosis D. Turner syndrome E. Von-Hippel-Lindau disease

Major takeaway: An increased risk of acute leukemia is associated with several genetic syndromes, including Down syndrome, Klinefelter syndrome, Li-Fraumeni syndrome, Neurofibromatosis type 1, and Fanconi anemia. Main explanation While acute leukemia in childhood is not generally thought to be a familial disorder, several genetic syndromes are associated with an increased risk. These include Down syndrome, Klinefelter syndrome (XXY male), Li-Fraumeni syndrome (p53 mutation), Neurofibromatosis type 1, and Fanconi anemia. Some other leukemia risk factors are immune disorders (Bloom syndrome, ataxia-telangectasia, Wiskott-Aldrich), radiation exposure, chemotherapy exposure for a different cancer, and immune system suppression.The most common presenting symptoms are nonspecific: fever, infection, bleeding, bone pain, or lymphadenopathy. Other important symptoms include easy bruising, petechiae, or purpura. Blood testing often shows anemia and/or thrombocytopenia with either leukocytosis or leukopenia. Peripheral smear will show lymphoblasts. Classification of leukemia is based on immunohistochemistry and cytogenetics performed on lymphoblasts or myeloblasts (in the case of acute myeloid leukemia) derived from bone marrow biopsy.

A 60-year-old postmenopausal woman comes to the office because of fatigue, mild jaundice, and tingling in her lower extremities. Laboratory studies show elevated serum concentrations of homocysteine and methylmalonic acid. Complete blood cell count shows mild thrombocytopenia. Which of the following is most likely revealed upon peripheral blood smear for this patient? A. Megaloblastic anemia with hypersegmented polymorphonuclear leukocytes B. A microcytic anemia with hypersegmented polymorphonuclear leukocytes C. microcytic hypochromic anemia with decreased serum iron and increased total iron-binding capacity Show explanation D. A normochromic normocytic anemia with decreased serum iron and decreased total iron-binding capacity Show explanation E. Normal peripheral blood smear with normal serum iron and normal total iron binding capacity

Pernicious anemia is caused by the failure of gastric parietal cells to produce sufficient Intrinsic Factor to permit vitamin B12 absorption. Main explanation The symptoms and laboratory findings suggest a chronic progressive anemia caused by a failure to absorb vitamin B12 due to *a lack of available intrinsic factor (IF).* Autoimmune gastritis/pernicious anemia is associated with two forms of auto-antibodies: (1) antibodies directed against the transmembrane proton pumps of parietal cells (the cells that secrete gastric acid and intrinsic factor), and (2) antibodies directed against IF itself. The latter antibodies can prevent either the binding of vitamin B12 to IF, or the binding of the vitamin B12-IF complex to its receptor in the distal ileum (where absorption takes place).Serum levels of homocysteine and methylmalonic acid can be elevated, and complete blood cell count can show a thrombocytopenia. Vitamin B12 deficiency anemia is a megaloblastic anemia and is associated with hypersegmented neutrophils. In the past, pernicious anemia was often diagnosed with the Schilling test, in which patients were given radiolabeled vitamin B12 orally, and absorption was monitored by the amount of radioactivity appearing in the urine. Currently serological tests directed against IF antibodies are more commonly used. B. Microcytosis with hypersegmented neutrophils could be seen in iron-deficiency anemia or the anemia of chronic disease. Hypersegmented neutrophils, however, are most classically associated with vitamin B12 deficiency, which is strictly megaloblastic. D. This is anemia of chronic disease- traditionally encompassed any inflammatory, infectious, or malignant disease

A 27-year-old woman comes to the office because of intermittent episodes of bloody diarrhea and abdominal pain. She also reports that she noticed a lesion on her left shin. She initially believed that she must have bruised it, but she then noticed that a painful ulcer began to form quite rapidly. Which of the following is the most appropriate step in management? A. Perform a biopsy B. Wide surgical debridement C. Prescribe a corticosteroids D. Prescribe an antibiotic E. Prescribe warfarin

Pyoderma gangrenosum is a neurophilic dermatosis that causes painful skin ulceration, usually of the lower legs, and may be triggered by an underlying disease. The best management of pyoderma gangrenosum is treatment of the underlying disease or anti-inflammatory agents. Main explanation Pyoderma gangrenosum is an uncommon cause of very painful skin ulceration. It may affect any part of the skin, but the lower legs are the most common site. Pyoderma gangrenosum is classified as a neutrophilic dermatosis and may be a reaction to a systemic disease or condition such as inflammatory bowel disease, rheumatoid arthritis, myeloproliferative disorders, or others. The best management is the treatment of the underlying disease, which in this case is ulcerative colitis. Treatment with oral corticosteroids as well as sulfasalazine will address the underlying bowel inflammation in this patient. Topical and systemic corticosteroids may be helpful in treating pyoderma gangrenosum regardless of underlying disease. E. Pyoderma gangrenosum is not caused by an infectious agent. Unless there is a secondary infection in the ulcer, antibiotics would not be helpful.


Kaugnay na mga set ng pag-aaral

Susan Wilson (2): A 26-year-old primigravida during labor and delivery

View Set

Week 3: Evaluating & Selecting Alternatives

View Set